Вы находитесь на странице: 1из 84

1.

Which of the following is the most likely diagnosis in a demented 70 y/o man
with high intensity on T2-weighted images and focal lesions in the periventricular
and subcortical white matter?

a. Multiinfarct dementia
b. Picks disease
c. Multiple sclerosis
d. Alzheimers

Type- Neuro
Answers- a
Notes-

by KW
Verified Osborn p774
1994 Single Best Answer Question
Changes described above likely represent changes of small vessel ischemic disease
and therefore multi infact dementia.

2. Which of the following is the most likely diagnosis in a head trauma patient
who loses consciousness, recovers to have a lucid interval, and again loses
consciousness?

a. contusion
b. subdural hematoma
c. subarachnoid hemorrhage
d. epidural hematoma

Type- Neuro
Answers- d
Notes-

1992 Single Best Answer Question


General guidelines following head trauma:
1) conscious ---> unconscious = nonspecific
2) unconscious + doesnt wake up = DAI
3) unconscious --> lucid --> unconscious = epidural
DAI injury is due to shearing and is most commonly seen in the gray-white
junction, the corpus callosum, and basal ganglia. (D p232-3).
by CM

3. Which of the following are dense on noncontrast CT?

a. CNS lymphoma
b. medulloblastoma
c. schwannoma
d. prolactinoma

Type- Neuro
Answers- a,b
Notes-

by KH
1995 Question
not verified

4. At what age do you first see the adult pattern of myelination on a 1.5 tesla
magnet?
a. 1 week
b. 8 months
c. 1 month
d. 2 years
e. 4 years

Type- Neuro
Answers- d
Notes-

by AE
1997 Question

5. Which of the following is true regarding intraventricular meningioma?

a. in results in increased CSF production


b. it frequently seeds the CSF
c. it obtains its blood supply from the external carotid artery
d. it most commonly involves the atrium of the lateral ventricle

Type- Neuro
Answers- d
Notes-

Approximately 2% of intracranial meningiomas have no dural attachment. These


tumors arise form choroid plexus stromal cells or the tela choroidea and grow as
intraventricular masses. They are usually confined to the ventricles but
occasionally penetrate into adjacent brain. osborn588; Intraventricular (2-5%);
80% in lateral (l>r), 15% in 3rd, 5% in 4th ventrile; from infolding of meningeal
tissue during formation of choroid plexus. Most common trigonal intraventricular
mass in adulthood! D250.
1993 Single Best Answer Question
Intraventricular meningiomas typically occur around the choroid plexus in the
trigone of the lateral ventricle and have a distinct propensity for the left
lateral ventricle. They calcify in 45-68% of cases. Blood supply to an
intraventricular meningioma is usually via choroidal
vessels.
by KH

6. Know these venous structures of the brain:

a. vein of Galen
b. internal cerebral vein
c. inferior sagittal sinus
d. basal vein of rosenthal

Type- Neuro
Answers-
Notes-

by KH
1995 Question
not verified

7. Which of the following is true regarding thyroglossal duct cysts?

a. Papillary carcinoma is the most common tumor to arise in these cysts


b. Most often superficial to the strap muscles
c. Commonly demonstrate thick rim enhancement
d. Most often septated

Type- Neuro
Answers- a
Notes-

D332
Thyroglossal duct cysts on CECT appear as a low density mass with a uniformly thin
peripheral rim of enhancement. Septations occasionally are seen. located at or
below the level of the hyoid bone are embedded in the strap muscles adjacent to
the outer margin of the thyroid cartilage. Unlike, the thyroglossal duct cyst,
other lesions found in this area are superficial to the strap muscles. Papillary
carcinoma is the most common tumor to arise from these cysts(Head & Neck Imaging-
Som p532-3).
Thyroglossal duct cysts are midline masses lined by squamous cell mucosa;
suprahyoid 20%, hyoid 15%, infrahyoid 65%. Usually a midline / paramediancystic
mass 2-4cm with infrahyoid strap muscles beaking over the edge of the cyst.
Complications include infection, and duct carcinoma (<1%). (D254)
They appear on MR scan as well-circumscribed fluid-density structures in the
midline. (TF)
Thickening and enhancement of the wall indicates infection.
AJNR Am J Neuroradiol 2000 Apr;21(4):770-4
Thyroglossal duct carcinoma should be suspected in an adult patient in the
presence of a solid nodule or invasive features in association with a thyroglossal
duct lesion visible on CT scans or MR images. The presence of calcification, which
is seen best on CT scans, may be a specific marker for carcinoma.
Arch Pathol Lab Med 2000 Jan;124(1):139-42
The incidence of papillary thyroid carcinoma arising in a thyroglossal duct cyst
is rare and occurs in about 1 % of thyroglossal duct cysts. Only 17 such cases
diagnosed with fine-needle aspiration biopsy have been previously reported in the
English-language literature, with a diagnostic rate of 53%.
Radiographics 1999 Jan-Feb;19(1):121-46; quiz 152-3
Cervical congenital cystic masses constitute an uncommon group of lesions usually
diagnosed in infancy and childhood. The most common congenital neck mass is the
thyroglossal duct cyst. The diagnosis is easily established from the presence of a
cystic lesion in the anterior midline portion of the neck. The vast majority of
branchial cleft cysts arise from the second branchial cleft. They can occur
anywhere from the oropharyngeal tonsillar fossa to the supraclavicular region of
the neck. Cystic hygroma is the most common form of lymphangioma. In the neck,
cystic hygromas are most commonly found in the posterior cervical space. They
typically extend into adjacent structures without respecting the fascial planes.
Dermoid and epidermoid cysts result from sequestration of ectodermal tissue. The
floor of the mouth is the most common location in the neck. Cervical thymic cysts
are very uncommon lesions and are found anywhere from the angle of the mandible
down to the sternum. Laryngoceles are classified into internal, external, and
mixed types and have a frequent association with laryngeal carcinoma.
1998 Board Question
by CM

8. Metachromatic leukodystrophy:

a. inherited metabolic disorder


b. acquired secondary to enviornmental factors
c. secondary to ischemia

Type- Neuro
Answers- a
Notes-

by KW
Dahnert p252, Osborne 722
Most common hereditary leukodystrophy (demyelinating disease)
Autosomal recessive lysosomal disorder
Bilateral symmetric demyelination with sparing of the sub-cortical U fibers
1997 Old Board Question

9. Which of the following modalities is most useful to distinguish recurrent


glioblastoma from radiation-induced necrosis?

a. contrast-enhanced CT
b. mass effect on CT
c. gadolinium-enhanced MR
d. 18F-deoxyglucose PET scan

Type- Neuro
Answers- d
Notes-

by KW
Verified Osborn p550
1993 Single Best Answer Question

10. Which of the following represent the OMU complex sinus pattern?

a. Sphenoid only
b. Frontal only
c. Ethmoid only
d. Maxillary only
e. Frontal, anterior ethmoid, and maxillary

Type- Neuro
Answers- e
Notes-

The osteomeatal complex is the control point for drainage of all 3 sinuses:
frontal, anterior ethmoid, maxillary (P572).
1999 Board Question
by MH

11. Which of the following structures is the most medial structure in the
cavernous sinus?

a. optic nerve
b. trigeminal nerve
c. cranial nerve VI
d. cranial nerve IV

Type- Neuro
Answers- c
Notes-

Dahnert p.203
Som p880,Osborne p463
1995 Single Best Answer Question
In the cavernous sinus lie the CNIII- which is in the superolat portion, CNIV-
which lies just below III, V-1, V-2, VI- which lies just lateral to the carotid. p
51 Requisites
The optic nerve is not in the cavernous sinus.
The most medial according to the fig 641 in Clemente is III, the oculomotor nerve,
then abducens-VI.
ref: Requisites p 51, Clemente fig 641
by AE

12. Which of the following tumors cannot be diagnosed with MIBG?

a. Pheochromocytoma
b. Meningioma
c. Carcinoid
d. Renal Cell CA
e. Medullary carcinoma of the thyroid

Type- Neuro
Answers- b,d
Notes-

(Metaiodobenzylguanidine is a guant=ethidine analog similar to norepinephrine. It


is taken up by chromaffin cells and is therefore useful for imaging normal and
abnormal sympathetic adrenergic tissue, especially pheochromocytomas, whether
located in the adrenal medulla or ectopically, and neuroblastomas. MIBG is
localized in other neuroendocrine tumors to lesser degree, including carcinoid,
medullary thyroid carcinoma, and paraganglioma. In the setting of
pheochromocytoma and neuroblastoma, the sensitivity and specificity of MIBG are
high, approaching 90%. mettler365.
C or d. Flawed question? MIBG (metaiodobenzylguanidine) is recognized as a
neurotransmitter precursor in any neuroectodermal tumor (carcinoid, Neuroblastoma,
pheochromocytoma, and paraganglioma) and normal activity in the liver, spleen,
salivary glands, and myocardium. Uptake of free iodine by the thyroid is blocked
by premedication with Lugols solution. Imaging performed 1, 2, 3 days after
administration. Absent uptake in the adrenals is normal; bilateral increased
uptake is hyperplasia; unilateral increased uptake is suspicious for a
neuroectodermal tumor; distant metastasis are also detectible. (P841) Medullary
carcinoma part of MEN.
1998 Board Question
by MH

13. Which of the following would be seen by MR in a patient who is 1 day S/P CVA?

a. vascular enhancement
b. mass effect
c. meningeal enhancement
d. parenchymal enhancement
e. hemosiderin deposition

Type- Neuro
Answers- a,b
Notes-

Dahnert p244
Meningeal and parenchymal enhancement at 1-7 days
Arterial enhancement has proven useful as an early marker of infarction.
Enhancement in arteries appears to be more sensitive that T2WI in the first few
hours of acute infarction, and thus enhancement is clearly useful for the workup
of acute ischemic syndromes. because normal rapid arterial flow produces a flow
void, arterial enhancement represents slow flow, and this can be detected in
approximately 50% of ischemic lesions. neuro requisties 112
1995 True/False Question
Findings on MR may be seen within a few hours. Swelling is observed on T1 and
proton density w/o changes of signal. 8hrs, high signal develops on T1 and PDWI.
16hr, low signal on T1. Arterial enhancement which is slow flow is detected in
approximately 50% of ischemic lesions. Arterial enhancement is more sensitive than
T2 in the first few hr of stroke. Reappearance of flow void occurs around 7-
11days. this suggests establishment of collateral flow of lysis of embolus.
Noncortical infarctions have parenchymal enhancement between 4-7 days whereas
cortical infarctions enhance after 6 days. Enhancement of the infarct may last 6-
8weeks. On CT, 6hr-faint loss of gray-white border, 12-24hr, indistinct low
density, feeble mass effect, >24hr, more mass effect, 3-5days, peak mass effect.
Parenchymal enhancement is rarely
seen before 3 days. If see earlier enhancement, either because of excellent
collaterals, or embolus has moved distally. Mass effect resolves at around 2-4
weeks. ref: Requisites p 112
by KH

14. Which one of the following infections of the brain is most commonly seen in a
patient with AIDS?

a. CMV encephalitis
b. HIV encephalitis
c. toxoplasmosis
d. cryptococcus

Type- Neuro
Answers- b
Notes-

Primer p514
Toxoplasma is the most common opportunistic CNS infection in AIDS patients.
Toxo is the most common focal CNS infection in AIDS.
1995 Single Best Answer Question
by KH

15. Which one of the following vessels is affected in the Wallenberg (lateral
medullary) stroke syndrome?

a. branches off the basilar artery


b. ipsilateral PCA
c. ipsilateral PICA
d. contralateral PICA
e. contralateral AICA

Type- Neuro
Answers- d
Notes-

Osborne p.368
1992, 1988 Single Best Answer Question
by AE

16. Which tumor is most likely to have drop mets:

a. Medulloblastoma
b. Ependymoma

Type- Neuro
Answers- a
Notes-

by KW
Dahnert p172
Up to 33% of Medulloblastomas have drop mets
Check Osborne
2000 Board Question

17. Which of the following associations are true:

a. Sarcoidosis - uveitis
b. None of the above
c. Sjogrens - keratoconjunctivitis

Type- Neuro
Answers- a,c
Notes-

Not Verified
1993 Board Question

18. Which artifact may be seen adjacent a vessel in spinal MRI?

a. Ghosting
b. Chemical shift
c. Phase misregistration

Type- Neuro
Answers- a
Notes-

Not Verified
1993 Board Question

19. A patient presents with pectoralis weakness and numbness of the skin over the
lateral shoulder. Which nerve is damaged?

a. Radial
b. Musculocutaneous
c. Axillary

Type- Neuro
Answers- c
Notes-

Lateral cutaneous branch of the axillary nerve provides sensation to lateral


shoulder
Not Verified
1996 Question

20. Wormian bones can be seen with all of the following except:

a. Chiari II malformation
b. Cleidocranial dysostosis
c. Osteogenesis imperfecta
d. Normal variant

Type- Neuro
Answers- a
Notes-

Dahnert p.145, Chiari II-> Luckenschadel skull in 85%


d. Wormian bones are intrasutural ossicles in the lambdoid, posterior saggital,
and temperosquamosal sutures which are normal up to 6 months of age. Mnemonic:
PORK CHOPS I: pyknodysostosis, osteogenesis imperfecta, rickets (in the healing
phase), kinky hair syndrome, cleidocranial dysostosis, hypothyroidism /
hypophosphatasia, otopalatodigital syndrome, primary acro-osteolysis (Hajdu-
Cheney) / pachydermoperiostosis / progeria, syndrome of Down, idiopathic.
1998 Board Question
by AE

21. You need to know what amaurosis fugax was:

a. Embolus to the internal carotid artery


b. Embolus to the right ophthalmic artery

Type- Neuro
Answers- b
Notes-

amaurosis fugax (=transient premonitory attack of impaired vision due to retinal


ischemia) in 12% caused by transient hypotension or emboli of platelets/
cholesterol crystals which may be revealed by funduscopy. d183.
1989 Board Question
by KH

22. 29 year old female with acute vision loss and headache. MR shows a
sellar/super sellar lesion with fluid levels. The most common cause is likely:

a. Ependymoma
b. Pinealoma
c. Pituitary apoplexy
d. Craniopharyngioma

Type- Neuro
Answers- c
Notes-

by KW
Dahnert p266
Pituitary apoplexy = massave hemorrhage into pituitary adenoma with sudden
infarction of pituitary gland
Patients present with headache, nausea, vomiting, acute visual defects
Sheehan syndrome = pituitary infarct of anterior pituitary gland post-partum
1997 Old Board Question

23. A 20 y/o was weight lifting and suddenly developed confusion. He has
diminished right visual field response. The left pupil is smaller than the right.
He is speaking gibberish. His brother has been nursing a cold sore for 10 days.
Which of the following is the most likely diagnosis?

a. left internal carotid artery dissection


b. herpes encephalitis
c. PICA distribution infarct

Type- Neuro
Answers- a
Notes-

Verified Osborne p234


1993 Single Best Answer Question
For an internal carotid dissection to do this, there must also be emboli since the
lesions
are separated in space (3rd cranial nerve nucleus, left parietotemporal region).
Sx for extracranial
dissection can present with neck and face pain, HA, horners syndrom , cranial
nerve
involvement, ischemic sx. With cervical internal carotid dissections, get Raeders
syndrome-
ptosis, miosis, HA, and preserved ipsilateral face sweating because of preserved
sympathetic
fibers. Dissection can occur after blunt trauma, abrupt head turning, sports
injury, direct
puncture. p 133-4 Req.
Right PICA syndrome includes right-sided ataxia, loss of pain and temperature
sensation
on the right side of the face, loss of pain and temperature sensation in the left
upper and lower
extremities, difficulty swallowing, loss of taste on the right, vertigo, and
nystagmus.
HSV-1 causes encephalities. Sx include alteration of pt state of consciousness
with
lethargy, drowsiness or stupor. Pt can have confusion, disorientation,
hallucinations. Pt can have
N/V, HA. Can get aphasia, involuntary movemnets, ataxia, sensory defects, loss of
retentive
memory. p 2033 Harrisons 12th ed.
by AE

24. A 23-year-old patient presents with headache for 6 weeks and occasional
vomiting without nausea. Physical examination reveals bilateral papilledema and
mild truncal ataxia. Which of the following imaging findings would be most
consistent with this history?

a. Obstructive hydrocephalus
b. Multiple infarcts
c. Massive subarachnoid hemorrhage
d. Low CT attenuation within the periventricular white matter

Type- Neuro
Answers- a
Notes-

Colloid cysts are smooth spheric cysts ranging in size from 0.3 cm to 4 cm. They
comprise approximately 2% of all intracranial tumors, and are most common in the
third to fifth decade with no sex predilection. Their location is nearly constant
in the anterior portion of the third ventricle with rare cases reported in the
lateral ventricles. Their characteristic location explains the clinical
presentation of episodic headache secondary to transient obstruction of the
foramen of Monro related to head position. More commonly, however, affected
patients present with gradual onset of symptoms related to obstructive
hydrocephalus. Such symptoms include insidious onset of headache, gait apraxia,
mental status changes, and papilledema. (Although d may be seen with interstitial
edema secondary to hydrocephalus, the best answer is c).
1999 Board Question
by CM

25. A 25 yr old male is involved in a skiing accident. The following day he


presents with unilateral headache and a right-sided Horners Syndrome. The most
likely diagnosis:

a. Internal carotid artery dissection


b. Vertebral artery dissection
c. Internal carotid artery occlusion
d. External carotid artery dissection

Type- Neuro
Answers- a
Notes-

b. Oculosympathetic paresis (Horners syndrome) in association with retro-orbital


headache is highly suggestive of ICA dissection.
1998 Board Question
by MH

26. A 30 y/o female had a severe headache 6 days ago. She has a known Berry
aneurysm and now presents with an acute headache and one episode of nausea without
vomiting. What is the most likely diagnosis?

a. communicating hydrocephalus
b. recurrent subarachnoid hemorrhage

Type- Neuro
Answers- b
Notes-

Berry aneurysm- due to congenital weakness in the media and elastica of arterial
wall, occurs at branching points
Aneurysm location: 1) Ant comm artery/ ant cerebral artery
2) ICA
3) Middle cerebral artery
4) Vertebrobasilar
1991 Single Best Answer Question
Intracranial aneurysms are the most common atraumatic cause of subarachnoid
hemorrhage. Only about 10% occur in the posterior fossa, the majority of these
arising in the
basilar tip. At least 20% are multiple. By definition, aneurysms larger than 2.5
cm are termed
giant. Although the vast majority of berry aneurysms occur in isolation, certain
diseases
predispose to their formation. These include autosomal dominant polycystic kidney
disease
(???6% incidence of aneurysm), AVM, fibromuscular dysplasia, coarctation of the
aorta, and
collagen vascular disease (Ehlers-Danlos and Marfans).
by AE

27. A 35 y/o male sustained blunt trauma to the neck two weeks prior to
development of amaurosis fugax and Horners syndrome. Which of the following is
the most likely diagnosis?

a. aneurysm of the internal carotid artery


b. pseudoaneurysm of the internal carotid artery
c. pseudoaneurysm of the vertebral artery
d. aneurysm of the vertebral artery

Type- Neuro
Answers- b
Notes-

by KW
Verified Osborn p234, Dahnert p314
Carotid dissection is a frequent cause of ipsilateral Horners syndrome and
amaurosis fugax
1993 Single Best Answer Question
aneurysm = focal dilatation of an artery
pseudoaneurysm = organized hematoma from a vessel that has bled.

28. A 37 year old female who had been painting for two weeks develops loss of pain
and temperature sensation within the left face and on the contralateral side of
her body. She also has right truncal ataxia and left tongue weakness. This is
most consistent with?

a. Wallenberg syndrome secondary to vertebral artery dissection


b. Horners syndrome secondary to internal carotid artery dissection
c. Miller-Gruber syndrome from a pontine glioma

Type- Neuro
Answers- a
Notes-

This sounds like Wallenberg (Lateral Medullary) Syndrome which is caused by


vertebral artery occlusion or PICA infarct (Osborn pp366-8). I couldnt find
Miller-Gruber syndrome.
Vascular dissection of the carotid or vertebral artery is an often-overlooked
cause of stroke in young patients. Dissection may occur following blunt trauma to
the neck, but also has been reported after abrupt head turning. There may be a
predisposition to this condition in persons with fibromuscular disease. MR can
detect the intramural hemorrhage and also note the residual patent lumen of the
vessel. The intensity characteristics of the intramural hemorrhage follow those of
the parenchymal hemorrhage. MR also is useful for following the dissection to note
both when the hemorrhage has been reabsorbed and when the normal lumen dimensions
are reestablished. The major complications of vascular dissection result from
embolic phenomena. Vascular dissection should be considered when stroke is present
in patients without significant risk factors. The category of atraumatic
dissections includes those that are spontaneous (i.e., without a recognized
precipitating cause) and those that are due to trivial trauma. The precipitating
event in coronary artery dissection due to trivial trauma often involves either
rapid motions of the head and neck, prolonged head turning, or neck extension.
Examples of these events include unusual head positions, strenuous exercise, and
vigorous vomiting, coughing, or sneezing. Chiropractic manipulation is another
well-recognized form of trauma that may cause coronary artery dissection. A
history of any of these precipitating events should be carefully sought in any
patient with symptoms or signs that may be referable to coronary artery
dissection. Possible mechanisms by which the ICA is injured in dissection due to
trivial trauma include stretching of the artery over the lateral mass of the C1
and C2 vertebral bodies, arterial compression between the angle of the mandible
and the upper cervical vertebral bodies, and injury by a prominent styloid process
(TF).
Oculosympathetic paresis (Horners syndrome) in association with retro-orbital
headache is highly suggestive of ICA dissection. (TF)
1999 Board Question
by CM

29. A 40 year old comes in with seizures. CT shows hypodensity in bilateral


temporal lobe and cingulate gyrus without any mass effect. Most likely cause?

a. Low-grade glioma
b. Herpes encephalitis
c. Mesial temporal sclerosis

Type- Neuro
Answers- b
Notes-

by KW
Herpes usally associated with mass effect, classic location for herpes however.
Mesial temp sclerosis usually unilateral, after long standing epilepsy
2000 Board Question

30. A 40 year old female presents with fullness of the cheeks and history of
progressive dry mouth. Imaging reveals bilateral enlarged parotid glands with
multiple cysts. The most likely etiology for this condition is?

a. Parotid adenomatosis
b. HIV
c. Warthin tumor
d. Sjogrens syndrome
e. Bilateral pleomorphic adenomas

Type- Neuro
Answers- d
Notes-

1999 Board Question


by MH

31. A 40 year-old male is involved in a skiing accident. He is initially


asymptomatic, but then develops Horners syndrome, left-sided facial pain, and
ptosis. Which of the following is the most likely cause?

a. carotid artery dissection


b. vertebral artery dissection
c. epidural hematoma
d. internal carotid artery occlusion

Type- Neuro
Answers- a
Notes-

thought if this was not single best answer format that vertebral artery dissection
might also cause these symptoms. Dissection of the cervicocerebral arteries cause
stroke in kids and young adults. Trauma
is the most frequent cause. Horners syndrome occurs in over half the cases. p
1981-2 Harrisons
Dissection can involve the carotid or vert. Sx include neck and face pain, HA,
Horners, CN involvement, ischemic sx that can occur days to weeks from
dissection. Ptosis, miosis, HA with preservation of ipsilateral facial sweating
(Raeders syndrome) can occur with cervical ICA dissection. Sympathetic fibers for
sweat go with ECA and those controling the dilator pupillae and superior palpebral
muscles follow the ICA . p133-4 Requisites
1994 Single Best Answer Question
by KH

32. A 45 y/o woman has unilateral papilledema and calcification of the junction of
the opticnerve and posterior globe. What is the most likely diagnosis?

a. Drusen
b. retinoblastoma
c. melanoma
d. choroidal osteoma

Type- Neuro
Answers- a
Notes-

D282 Neuro Req290


1988 Single Best Answer Question
by MH

33. A 45 year old female presents with midfacial trauma and signs of acute central
cord syndrome. Cervical spine films reveal normal alignment of the spine and pre-
vertebral soft tissue swelling. This most likely represents?

a. Hyperextension injury
b. Pillar fracture
c. Atlanto-axial dissociation
d. Hangmans fracture
e. Non-displaced odontoid fracture

Type- Neuro
Answers- a
Notes-

Many hyperextension injuries in the lower cervical region reveal few or no


radiographic abnormalities, even when the injury is unstable or results in severe
neurologic damage. resnick805
Per Dr. Mojtahedi. The acute central spinal cord injury syndrome may present when
damage involves principally the central region of the spinal cord. This syndrome
is characterized by disproportionately more motor impairment of the upper than
lower extremities, bladder dysfunction, and varying degrees of sensory loss below
the level of the lesion. (TF)
There is no bony interlocking at C1-2, and the transverse ligament-odontoid system
is the main stabilizing bond of the atlantoaxial complex. Additional support is
provided by the alar ligaments connecting the odontoid tip to the medial part of
the occipital condyles as well as the anterior and posterior longitudinal and the
accessory ligaments. Damage to these supporting structures results in C1-2
instability, and if severe, may cause the posterior arch of C1 to crush the cord
against the upward projecting dens with varying degrees of neurologic damage or
even death. There is about 10 of flexion-extension at C1-2, and a small amount of
lateral translation in lateral tilt. The main motion is rotary and accounts for
about 50% of total neck rotation. The pivot of this rotation is the eccentrically
placed odontoid; hence C1 rotates across the canal of C2, narrowing the space
available for the cord. To allow for this, and for some degree of forward
pathologic C1-2 displacement without cord pressure, the canal of C1 is large,
about 3 cm in anteroposterior (AP) diameter in the average adult, and is occupied
equally by cord, odontoid, and free space. The relationship has been characterized
by Steel as the rule of thirds . The cord moves into the free space when C1 is
displaced. If the displacement approaches 1 cm (the width of the odontoid) the
danger of cord damage increases, and the patient may not be able to withstand a
second episode of trauma without neurologic sequelae or even death. C 1 ring size
may vary from 25 mm to 35 mm (AP) to 23 mm to 30 mm lateral; cord and odontoid
size vary little; each is usually 1 cm in diameter. Hence, the individual with a
small C1 ring may be in greater danger of cord damage because there is less space
available for the cord. (TF)
1999 Board Question
by KH

34. A 52 y/o interior decorator has just finished 2 weeks of painting a house.
She presents with loss of pain and temperature sensation on the right side of the
body with loss of pain and temperature sensation on the left side of the face.
She also has ataxia and her tongue deviates to the right. What is the most likely
diagnosis?

a. glioma
b. central pontine infarct
c. Horners syndrome
d. Webers syndrome
e. Wallenberg syndrome secondary to left vertebral artery dissection

Type- Neuro
Answers- e
Notes-

by KW
Verified Osborn p368
Wallenberg = Lateral Medullary Syndrome
1995 Single Best Answer Question
vertebral artery distribution, the lateral medulla is most often affected +/ the
posterior inferior
cerebellum. Lateral medullary infarction can result from occlusion of vert, PICA,
sup, middle,
inf lateral medullary arteries. In 70-80% of cases, get after ipsilateral
vertebral artery occlusion.
Get pain, numbness, inpaired sensation over ipsilateral face, arm, trunk, leg AND
contralateral
impaired pain, thermal sense over half of body. Get ataxia of limbs, falling to
side of lesion,
nystagmus, horners, dysphagia, hoarsenes, paralysis of palate, vocal cords, and
diminished gag.
Deviation of the tongue to the right is from 12th nerve palsy on the right which
can result
from medial medullary syndrome -- occlusion of the vert or branch of the vert or
lower basilar
artery.
Webers syndrome is a result of PCA infarct Webers is a third nerve palsy with
contralateral hemiplegia. Involves infarction of the third nerve and cerbellar
peduncle. fig 351-6
p 1984
ref: Harrisons 12th ed p 1988-9, 1984, 1987
dissection b/c ipsilateral face sx and tongue deviation.

35. A 55 year old male undergoes brain MR. A temporal lobe lesion is identified
which is very dark centrally and bright peripherally on T2 weighted images. The
entire lesion is isointense on T1 weighted images. This most likely represents?

a. Lymphoma
b. Aggressive glioma
c. Hemorrhagic contusion
d. Abscess
e. Cavernous hemangioma

Type- Neuro
Answers- c
Notes-

a.This sounds like the single best answer to me although, Im not certain. The
temporal lobe is a common location for contusion(D p225-6). b.Cavernous
hemangioma (=cavernous angioma) has mixed signal intensity centrally surrounded by
a hypointense rim (hemosiderin) on T2W (Dahnert p221, Osborn p313). c.GBM may
show central necrosis and hemorrhage of varying age (dark on T2W) with peripheral
edema (bright)(Osborn p543). c.F (Osborn p620-3). e.F(Osborn p691).
1999 Board Question
by CM

36. A 7 year old patient with scoliosis and bilateral lower extremity weakness and
leg length discrepancy (left greater than right). The patient also has a sacral
dimple with a hairy patch. This is most consistent with?

a. Tethered cord
b. Ependymoma of the conus medullaris
c. Meningocele
d. Chiari II with lipomeningocele
e. Astrocytoma

Type- Neuro
Answers- a,c
Notes-

Dahnert p173,179
A tethered spinal cord is one that is held down in an abnormally low position by a
fibrous band, bone spur, or mass, which presumably has prevented it from ascending
to the normal level during embryogenesis. Tethering has significance when the cord
becomes stretched between (1) the tethering band, bone spur, or mass and (2) fixed
attachments along the dentate ligaments and at the foramen magnum. Stretching the
spinal cord constricts its blood supply, so the cord blanches and becomes
ischemic. The typical symptoms of tethered cord - paraparesis and paraplegia,
sensory loss, and reduced bladder and bowel function - are believed to result from
chronic repetitive cord ischemia. Spinal flexion and gain in height exacerbate the
stretching, so patients frequently have greater symptoms and present to medical
attention following unusual exercise or during growth spurts. Perhaps the most
straightforward example of tethered spinal cord is the tight filum terminale
syndrome. Tight filum terminale syndrome denotes traction on the conus medullaris
as a result of an abnormally short, abnormally thickened ilium terminale. This
term specifically excludes cases with concurrent diastematomyelia, fibrous
adhesions, or other cause for tethering of the cord. Females (57%) are affected
slightly more commonly than males. Patients typically present at ages 5 to 10
years or 10 to 15 years during periods of rapid gain in height. The presenting
symptoms include motor weakness (76%), pain (42%), bladder dysfunction (35%),
sensory loss (21%), and scoliosis (17 .5%). Fifty percent of patients exhibit
cutaneous stigmata, usually a midline skin dimple, less commonly a capillary
hemangioma or patch of hypertrichosis. Kyphoscoliosis is present in 17% to 25% of
cases. In one third of cases, the scoliosis improves after surgical section of the
ilium releases the spinal cord. Plain spine radiographs (nearly) always
demonstrate malformation(s) of the neural arches at one or more lumbar,
lumbosacral, or sacral levels. Most commonly these are incomplete fusion (spina
bifida occulta) of L-4, L-5, and/or S-1. Since every one of their 86 patients with
this syndrome had such malformed neural arches, Hendrick and co-workers suggest
that normal spine radiographs nearly exclude this diagnosis. Myelography, computed
tomography (CT), and magnetic resonance imaging (MRI) show that the tip of the
conus medullaris lies below L-2 in 86% of cases. In 10% to 15% of cases, the
spinal cord continues inferiorly without distinct termination and attaches to the
distal most thecal sac by a small lipoma. By definition, the filum is thicker than
2 mm in 100% of cases. It contains a filar fibrolipoma in 29% of cases.
1999 Board Question
by MH

37. A cerebral angiogram reveals an enlarged tentorial branch arising from the
meningohypophyseal trunk. This most likely represents?

a. Cerebellar metastasis
b. Dural AVM
c. Glomus jugulare
d. Hemangioblastoma
e. Vestibullar schwannoma

Type- Neuro
Answers- b
Notes-

by KW
Osborne p124 - enlarges to supply dural AVMs or neoplasms that involve the
tentorium, cavernous sinus, or clivus
Not Glomus tumor, Som p1074
The meningohypophyseal artery arises from the precavernous or intracavernous
segment of the internal carotid artery and has three major branches: tentorial,
inferior hypophyseal, and dorsal meningeal branches. The inferior hypophyseal
branch courses superiorly and medially and reaches the lateral surface of the
hypophysis. It divides into superior and inferior branches, which run in the
sulcus between the anterior and posterior pituitary. The tentorial branch
supplies the free margin of the tentorium. The dorsal meningeal artery courses
medially and posteriorly and supplies the dura over the clivus and the dorsum
sellae. Rich anastomoses are noted among the distal branches of the right and left
meningohypophyseal arteries.
1999 Board Question

38. A cervical spine myelogram is performed and the cord appears enlarged in the
AP projection but is small to normal in the lateral projection:

a. Hematomyelia
b. cervical spondylosis
c. ependymoma
d. Arnold-Chiari malformation with hydromyelia
e. epidural hematoma

Type- Neuro
Answers- b
Notes-

a.Epidural hematoma of the spine may cause cord compression (Osborn p870,872).
b.60% of intracranial ependymomas are located in the posterior fossa and 40% are
found above the tentorium. 90% of infratentorial ependymomas occur in the 4th
ventricle. Ependymomas are known for there tendency to ooze or extrude down
through the foramen magnum into the upper cervical spine behind the
cervicomedullary junction (Osborn p567,p569 fig13-53).
c.Spondylosis with osteophytosis can cause spinal stenosis. (you would think
osteophytes would be seen during a myelogram)?T/F (Osborn p845).
d.Arnold-Chiari = Chiari II malformation. Hydromyelia causes an enlarged cord (I
would assume in AP & Lat. dimensions).
e. Hematomyelia = hemorrhage within the spinal cord.
by CM
1996 Question

39. A child is found to have a cystic cerebellar lesion with an enhancing mural
nodule. This most likely represents?

a. Choroid plexus papilloma


b. Medulloblastoma
c. Astrocytoma
d. Brainstem glioma
e. Ependymoma

Type- Neuro
Answers- c
Notes-

Verifed Osborne554
Pilocytic astrocytomas represent only 5% to 10% of all cerebral gliomas but
account for nearly one third of pediatric glial neoplasms. Opticochiasmatic-
hypothalamic PAs are one of the most common supratentorial neoplasms in children,
and cerebellar astrocytomas are the most common posterior fossa tumor in this age
group (in some series medulloblastoma is the most common pediatric infratentorial
tumor). Brainstem gliomas are the third most common pediatric infratentorial
tumor. osborn555
Cerebellar astrocytoma is the second most common tumor of the posterior fossa in
children 10-20%. Classically a cystic lesion with a tumor nodule (mural nodule)
in the cyst wall (50%), solid mass with cystic center 40-45%. Calcifications 20%.
(D168)
Ependymoma is 15% of pediatric posterior fossa tumors. Majority are benign, slow-
growing tumors of mature well-differentiated ependymal cells. Location: floor of
fourth ventricle (70%) of all intracranial ependymomas. CT: sharply marginated
multilobulated iso-slightly hyperdense 4th ventricular mass with heterogeneous to
moderately uniform enhancement of the solid portions (80%). (D176)
Medulloblastoma is the most common neoplasm of the posterior fossa in children 30-
40%. Classic features in 53%: 70% hyperdense, rapid intense homogeneous
enhancement 97%. Atypical features: cystic/necrotic areas 10-16%, calcifications
13%. May metastasize to subarachnoid with drop mets. (D190)
Brainstem glioma is 20-30% of all infratentorial brain tumors, 12-15% of all
pediatric brain tumors. Usually an anaplastic astrocytoma / GBM. Location: pons
> midbrain > medulla. CT: iso / hyopdense mass with indistinct margins, absent /
minimal / patchy contrast enhancement (50%), ring enhancement in necrotic /
aggressive tumor, engulfment of basilar artery. (D179)
Choroid plexus papilloma 2-5% of all brain tumors in children. Location: glomus
of choroids plexus in trigone of lateral ventricles L > R (in children). Large
mass with smooth lobulated border and small foci of calcifications. Distinctive
feature: engulfment of the glomus of the choroids plexus. Ventricular dilatation
secondary to overproduction / decreased resorption of CSf. CT: iso- to mildly
hyperdense with intense homogeneous enhancement with contrast. (D172)
1999 Board Question
by KH

40. A child is found to have a homogeneously enhancing mass lesion arising from
the cerebellar vermis and bulging into the 4th ventricle. This most likely
represents?

a. Ependymoma
b. Hemangioblastoma
c. Brainstem glioma
d. Astrocytoma
e. Medulloblastoma

Type- Neuro
Answers- e
Notes-

Req p.83-86, Medulloblastoma- usually arise in the midline from the cerebellar
vermis, usually compress the fourth
vent causing hydrocephalus, moderate enhancement.
-Ependymoma- enlarge the fourth vent, 40-50% calcify, mild enhancement
-Hemangioblastoma- most common primary infratentorial tumor in adults, assoc. with
polycythemia
PRIMITIVE NEUROECTODERMAL TUMOR [PNET, MEDULLOBLASTOMA]: Histologically, a PNET is
a malignant, embryonal childhood tumor arising in the cerebellum that is composed
of densely packed cells with round to oval, or carrot-shaped nuclei and scanty
cytoplasm. The cells have variable mitotic activity. The PNET usually arises in
the cerebellum and is centered in the midline. The mass is homogeneous and
spherical in shape, growing as if a solid expanding ball. Hemorrhage, cysts, and
calcifications occur in approximately 10% of cases. The residual lumen of the 4th
ventricle may be seen as a crescent of CSF density or intensity along the anterior
surface of the mass. On CT, the tumor frequently appears of homogeneous
hyperattenuation relative to gray matter. The increased attenuation is thought to
be due to the dense cellular nature and high nuclear to cytoplasm ratio of its
cells. On MR, the mass is homogeneously hypointense on T1- and usually hypo- or
isointense on T2-weighted images relative to gray matter. By either CT or MR, the
PNET homogeneously enhances following intravenous contrast injection. Treatment
of PNET is complete surgical resection followed by aggressive chemotherapy and/or
radiation therapy. A complete metastatic work-up is needed as up to 30% of
patients with PNETs have leptomeningeal spread of tumor at the time of diagnosis.
Currently, the 5-year survival rate is approximately 45% in children younger than
8 years old at diagnosis and 74% in children older than 11 years old at diagnosis
(TF).
1999 Board Question
by AE

41. A common carotid injection demonstrates occlusion of the internal carotid


artery immediately above the bifurcation with subsequent filling of the
ipsilateral intracranial internal carotid artery. This most likely represents?

a. Collaterals via the opthalmic artery


b. Collaterals via the posterior communicating artery
c. Collaterals via the anterior communicating artery
d. An intact Circle of Willis
e. Previous ECA-ICA bypass

Type- Neuro
Answers- a
Notes-

The opthalmic arterys branches anastomosis with the ECA(Osborn p124,121).


by CM
1996 Question

42. A CT of the neck is performed to evaluate a patient with left vocal cord
paralysis. Inferior images should extend to?

a. Cricoid cartilage
b. AP window
c. Costo-phrenic angle
d. Thyroid gland
e. Thoracic inlet

Type- Neuro
Answers- b
Notes-

The left recurrent laryngeal nerve branches from the vagus (X) at the level of the
aortic arch and courses medially to the esophagus and superiorly back to the
larynx. The right recurrent laryngeal nerve branches from the vagus (X) at the
level of the subclavian artery and courses lateral to the esophagus superiorly
back to the larynx. netter68
1999 Board Question
by KH

43. A diabetic patient has a CT which shows enhancement in the inferior frontal
lobes. Which of the following is the most likely diagnosis?

a. infarct
b. herpes
c. lymphoma
d. fungal encephalitis
e. PML

Type- Neuro
Answers- b
Notes-

1993 Single Best Answer Question


Progressive multifocal leukoencephalopathy (PML) is a demyelinating disease with
viral etiology (papovavirus). It is associated with immunosuppression. It can
occur anywhere in the brain (usually in the subcortical white matter, but can
involve the cortex - in contrast to HIV encephalitis which is characteristically
periventricular). It can be solitary or multifocal. It is low intensity on T1
and high intensity on T2. Usually there is no enhancement.
Lymphoma is not uncommonly seen in immunosuppressed patients (AIDS, post-
transplant, etc.). There is a marked predilection for the basal ganglia,
cerebellar hemispheres, thalamus, brain stem, corpus callosum, and subependymal
region. Primary CNS lymphoma occuring in AIDS patients is a highly aggressive
malignancy which is frequently necrotic and surrounded by vasogenic edema. The
central necrotic portion is hypodense on CT and the periphery enhances with
contrast. In contrast, lymphoma occuring in patients who do not have
AIDS is more typically a solid, hypercellular tumor and is therefore often
hyperdense on noncontrast CT scans. Following gadolinium or iodinated contrast,
enhancement is typically uniform. Primary CNS lymphoma virtually never calcifies.
Fungal encephalitis: Mucormycosis affects patients with abnormalities of host
defenses, particularly patients with diabetes and ketoacidosis. The fungus is
usually inhaled and rapidly destroys nasal mucosa. It may then spread into
paranasal sinuses (with or without bone destruction), orbit, skull base, or may
extend throught the cribriform plate and involve the anterior cranial fossa.
There is a high frequency (50% in some series) of intracranial extension. With
intracranial extension, there are low density abnormalities in the anterior
cranial fossa (these may in fact be present in any part of the brain). One can
also see mass effect and enhancement.
Herpes virus Type I is responsible for encephalitis in adults. One-third of cases
are due to primary infection (usually <18 y/o) and 2/3 are due to reactivation.
Herpes has a predilection for temporal lobes, insula, subfrontal region, and
cingulate gyrus. Involvement may initially appear unilateral but is typically
followed by less severe contralateral disease. This sequential bilaterality is
highly suggestive of herpes simplex 1 encephalitis (Osborn p695). MR examination
done during the first five days of infection shows high signal on PD and T2 in the
temporal and inferior frontal lobes with increasing mass effect. Linear,
subarachnoid, and gyriform enhancement may be seen.
by CM

44. A hemosiderin ring is:

a. brighter after the administration of gadolinium (T1 weighted images)


b. dark on T2 weighted MR
c. dense on non-contrast enhanced CT
d. bright on T1
e. best seen on CT

Type- Neuro
Answers- b
Notes-

1993 True/False Question


Hemosiderin rings are low density on CT.
Hemosiderin rings are dark on T1 and T2.
CT imaging of hemorrhage:
Acute (<1 week)- hyperdense [except with active bleeding(swirl sign), extreme
anemia(isodense), rebleed into old hematoma(fluid-fluid level), and coagulation
disorders(fluid-fluid level)]
Subacute (1 week-months) - isodense
Chronic (months-years) - hypodense
MR imaging of hemorrhage:
Hyperacute (hrs) - T1 isointense to dark, T2 bright (oxyHgb)
Acute (1-3days) - T1 isointense to dark, T2 dark (deoxyHgb)
Early Subacute(3d-1wk)-T1 bright, T2 dark (intracellular metHgb)
Late Subacute(1wk-months)-T1 bright, T2 bright (with dark rim)(free metHgb)
Chronic (months-yrs)-T1 dark, T2 dark (hemosiderin and ferritin)
by CM

45. A HNP at the L4-5 level produces symptoms at the:

a. S1 level
b. L4 level
c. L3 level
d. L5 level.
Type- Neuro
Answers- d
Notes-

Req p.448
1989 Board Question
by AE

46. A hyperdense mass with attenuation of 30-50 with surrounding well defined thin
enhancement:

a. Lymphoma
b. aging hematoma
c. abcess
d. high-grade glioma

Type- Neuro
Answers- b
Notes-

by KW
Dahnert p238,248
Lymphoma could have this appearance... can have little edema.... check Osborne
2000 Board Question

47. A lucent lesion in the calvarium with a sclerotic margin is most likely?

a. Hemangioma
b. Venous lake
c. EG
d. Epidermoid inclusion cyst
e. Metastatic disease

Type- Neuro
Answers- d
Notes-

by KW
Osborn 516
Dahnert p87,146
EG does not have sclerotic margin, Epidermoid does
Normal osteolytic lesion of the skull: emissary vein connecting inner and outer
skull; venous lake; pacchionian granulations (usually within 3cm of the SSS); and
parietal foraminae (embryonal rests).
Epidermoid inclusion cyst
1998 Board Question

48. A one year old is found to have strabismus and an enhancing mass of the sella:

a. Paraphyseal cyst
b. Craniopharyngioma
c. Optic glioma
d. Infundibuloma
e. Ectopic pinealoma

Type- Neuro
Answers- b
Notes-

by KW
Verified Dahnert p263,199,226
Pinealoma = Pineal Germinoma - Age 10-25 years, ectopic in sella, enhances
Craniophayngioma - most common suprasellar mass - Age from birth to seventh
decade, enhances, 2nd most common intrasellar mass
Optic glioma - Age 1st decade, peak ~ 5 years, slight enhancement
Infundibuloma and Paraphyseal cyst????
1996 Question

49. A patient has palsies involving cranial nerves III, IV, V1, V2, and VI. What
is the location of the lesion?

a. brainstem
b. superior orbital fissure
c. optic canal
d. cavernous sinus
e. inferior orbital fissure

Type- Neuro
Answers- d
Notes-

1994, 1993, 1988 Single Best Answer Question


Lesions in the cavernous sinus can affect: III, IV, V1, V2, and VI (Osborn p149).
Lesions in the superior orbital fissure can affect: III, IV, V1, VI (also the
orbital branch of the middle meningeal artery, the sympathetic nerve of internal
carotid plexus, the recurrent meningial artery, and the superior ophthalmic vein)
(Osborn p487, Dahnert p285).
Lesions in the optic canal can affect the optic nerve and the ophthalmic artery.
Lesions in the inferior orbital fissure can affect the infraorbital artery, vein,
and nerve (branch of V2)
Lesions in the brainstem can affect: cranial nerves I-IV if they arise from the
midbrain or above, V-VIII if they arise from the pons, and IX-XII if they arise
from the medulla.
by CM

50. A patient presents with a conductive hearing loss and ipsilateral facial nerve
palsy. This is most likely secondary to?

a. Menigioma at the cerebellar-pontine angle


b. Glomus jugulare
c. Transverse temporal bone fracture
d. Longitudinal temporal bone fracture
e. Glomus tympanicum

Type- Neuro
Answers- d
Notes-

The glomus tympanicum tumor is the most common tumor in the middle ear, followed
by the facial neuroma. Patients present with hearing loss, pulsatile tinnitus,
bruit, and reddish purple mass behind the tympanic membrane (D p322-3). CT will
demonstrate a characteristic nondestructive small soft-tissue mass protruding from
the cochlear promontory. Usually there are no other findings, except for
occasional fluid opacification. Contrast enhancement is difficult to define due to
CT beam hardening. MR may demonstrate enhancement with Gadolinium. Angiography
shows a small densely blushing middle ear mass. (TF)
Glomus jugulare tumors are usually larger and more destructive than tympanicum
tumors. They cause tinitus and hearing loss (D p323). Most follow a
characteristic pattern of growth. Pervasive destruction is a feature that
characterizes the invasiveness of the glomus jugulare tumor. The tumors commonly
expand the jugular fossa with irregular erosive changes. There may be involvement
of the occipital condyle, mastoid facial nerve canal, and hypoglossal canal.
Paragangliomas commonly extend through the jugular fossa into the inferior middle
ear cavity. They may spread along the dural sinuses toward the torcula or
inferiorly toward the chest. These tumors can erode the otic capsule, most
commonly near the cochlea. They may envelope the petrous carotid artery and extend
beyond the dura to occlude the jugular vein. Destruction of the caroticojugular
spine, best appreciated on axial CT is another characteristic finding with glomus
jugulare tumors. Although there may be mastoid and middle ear opacification, the
ossicles are rarely eroded or displaced. (TF)
Cerebellopontine angle tumor may cause cranial neuropathy including high frequency
hearing loss(VIII), tinnitus, facial motor(VII) and sensory dysfunction(V). (D
p193).
Temporal bone fractures have been divided into longitudinal and transverse types,
depending on the course of the fracture line. This classification tends to be
artificial in that fractures frequently extend in a serpiginous fashion. To some
extent, fractures more frequently extend along the plane of the temporal bone than
across it. Transverse fractures are perpendicular to the axis of the petrous
pyramid and run from the superior petrous ridge across the labyrinth usually to
the jugular fossa. These may cause facial nerve palsy and neurosensory hearing
loss. Longitudinal fractures are parallel to the axis of the petrous pyramid and
tend to involve the temporal squama with extension into the mastoid, reaching the
external auditory canal. Medially, the extent is usually anterior to the labyrinth
and is extralabyrinthian (TF). Longitudinal fractures cause conductive hearing
loss (no neurosensory loss) and may cause facial nerve palsy.
Peripheral facial nerve palsy may result from trauma. Facial nerve involvement
has been reported with both longitudinal and transverse fractures. The incidence
and morbidity is greater with the transverse variety. Mechanisms of injury
include displaced bone fragment, nerve transsection, and intraneural hematoma or
edema. The most severe damage to the nerve is at the meatal foramen (entrance of
the distal intracanalicular segment into the labyrinthine segment). (R-HN844).
(Dahnert p322).
1999 Board Question
by CM

51. A patient sustains a hyperextension injury to the cervical spine with


significant spinal cord injury. Radiologic findings which are more common
include:

a. Prevertebral soft tissue swelling


b. Dislocation of the facet joint
c. Comminuted fracture of the vertebral body with retropulsion of fracture
fragments into the spinal canal
d. Anterior/superior avulsion fracture of the vertebral bodies
e. Fractures through the pedicles

Type- Neuro
Answers- a
Notes-

Many hyperextension injuries in the lower cervical region reveal few or no


radiographic abnormalities, even when the injury is unstable or results in severe
neurologic damage. resnick805
1989 Board Question
by KH

52. A patient with spinal cord trauma has headache, diaphoresis, and blood
pressure fluctuations after intravesical instillation of 150 cc of contrast. All
symptoms are relieved by insertion of a Foley catheter. What is the level of
injury?

a. T8-9
b. cervical spine
c. S1-2
d. S3-4
e. T12-L4

Type- Neuro
Answers- b
Notes-

1992 Single Best Answer Question


Symptoms of autonomic dysreflexia occur with spinal cord lesions above T5 (level
of
greater splancnic nerve) secondary to alpha-adrenergic response below the spinal
cord lesion
uncontrolled by central centers. Therefore, there is a rapid rise in blood
pressure, pallor below
the level of the lesion, flushing and diaphoresis above the level of the lesion,
headache, coma,
and potentially death. Activation of the carotid sinus baroreceptors can cause
bradycardia.
Reference: Dahnert, p. 363.
by AE

53. A patient with unilateral field cut and medial rectus paralysis has a lesion
in:

a. cavernous sinus
b. optic chiasm
c. medulla oblongata and the pons
d. occipital lobe
e. cerebellopontine angle

Type- Neuro
Answers- a
Notes-

by KW
Dahnert 217
Not exactly sure what a unilateral field cut is, asssuming it is hemianopsia
involving one eye only.
This lesion is affecting the second and third cranial nerves. It must be along
the distal course.
A Cavernous sinus lesion (such as a aneurysm) can cause visual loss, oculomotor
loss, and trigeminal distribution pain.
1993 Single Best Answer Question
The lesion has to be before the optic chiasm. CNIII supplies extraocular muscles
except
lateral rectus and superior oblique. The nucleus is in the midbrain just
posterior to the red
nucleus and anterior and superior to the aqueduct. It goes thru the
interpeduncular cistern to
between the PCA and superior cerebellar arteries lateral to the PCOM. Thru the
caverous sinus
thru the superior orbital fissure. The optic nerve goes back to the lateral
geniculate in the
pulvinar of the thalamus and back thru the geniculocalcarine tract.
ref; Req p 45, see figure 2-14 p 44

54. A right paracentral L4/L5 disc herniation causes most likely causes which
nerve impingement:

a. S-1
b. right l5
c. none
d. right l4 and l5
e. right l4

Type- Neuro
Answers- b
Notes-

by KW
Multiple repeats...
2000 Board Question

55. A trauma patient has bilateral C6-7 facet dislocation and C6-7 cord level
findings with no sensory or motor function below that level. On the following day
he regains the bulbocavernous reflex. Which of the following is the best
explanation?

a. posterior cord syndrome


b. central cord syndrome
c. total cord syndrome
d. anterior cord syndrome
e. lateral cord syndrome

Type- Neuro
Answers- c
Notes-

per Stephan Haas


by KW
1994 Single Best Answer Question

56. A vertebral hemangioma most commonly will have which pattern of signal
intensity?

a. Bright on both T1 and T2


b. Dark on T1 and bright on T2
c. Dark on both T1 and T2
d. Bright on T1 and dark on T2

Type- Neuro
Answers- a
Notes-

The vertebral bodies and skull are most commonly involved and show dense vertical
striations. High signal intensity on both T1 and T2 (H67).
1999 Board Question
by AE

57. A young woman comes in with sudden onset of blurred vision. CT shows a
pituitary mass with suprasellar extension. This mass has area of high attenuation
and fluid level. Cause:

a. pituitary apoplexy
b. germinoma
c. sarcoidosis
d. metastatic disease
e. craniophyrangioma

Type- Neuro
Answers- a
Notes-

by KW
Dahnert p266
Pituitary apoplexy = massavie hemorrhage into pituitary adenoma with sudden
infarction of pituitary gland
Patients present with headache, nausea, vomiting, acute visual defects
Sheehan syndrome = pituitary infarct of anterior pituitary gland post-partum
2000 Board Question

58. About posttraumatic vertebral collapse, which is true:

a. The vertebral body becomes increasing dense


b. Involvement of the adjacent disc space
c. Development of large osteophytes
d. Juxtaarticular erosions

Type- Neuro
Answers- a
Notes-

by KW
2000 Board Question

59. All of the following are important in the evaluation of spinal stenosis
except?

a. Adjacent bony spurring


b. Posterior disc bulge
c. Hypertrophy of the dentate ligaments
d. The appearance of the spinal canal
e. Facet hyperostosis

Type- Neuro
Answers- c
Notes-

by KW
Osborn 845-848 does not mention dentate ligament....
Netter p155 lists the denticulate ligament as a small ligament connecting the cord
to the dura
Multiple sources do not list this ligament as a cause of spinal stenosis
All contribute to spinal stenosis except denticulate ligament hypertrophy. The
denticulate ligaments attach the cord to the dura mater at the nerve root exits.
(Netter)
1998 Board Question

60. An elderly male patient presents with slowly progressive cervical myelopathy.
Imaging reveals 5mm thick calcification/ossification within the anterior aspect of
the thecal sac immediately posterior to the C3-C5 vertebral bodies. This is most
consistent with?

a. Ankylosing spondylitis
b. Arachnoiditis
c. Ossification of the posterior longitudinal ligament
d. DISH
e. Calcification of a previous epidural hemorrhage

Type- Neuro
Answers- c
Notes-

by KW
Osborne p848
Ossification of the PLL is more common in Japenese, and found in mid-cervical and
mid-thoracic spine.
Ossification of the posterior longitudinal ligament is uncommon, occurring mostly
in the Japanese. Most common in the midcervical and midthoracic spine causing
neurologic symptoms (Osborn 1st ed. 848). AS causes ossification of the annulus
fibrosis (not the ALL (D27)) and a bamboo spine with biconvex intervertebral
disks, erosion of the anterior margins of the vertebral bodies and ossification of
the paraspinal ligaments (D113). Arachnoiditis causes blunting of nerve root
sleeves without cord displacement (D123). DISH (diffuse idiopathic skeletal
hyperostosis) occurs lower thoracic > lower cervical, occurring on the anterior
and right syndesmophytes of at least 4 contiguous bodies (D39).
1999 Board Question

61. Blood supply of glomus jugulare tumor:

a. occipital branch of external carotid artery


b. posterior auricular
c. dural branches
d. middle meningeal
e. ascending pharyngeal

Type- Neuro
Answers- a,b,e
Notes-

D p323
Som 1074
by CM
1996 Question

62. Brain MR reveals multiple bright areas of T2 signal intensity within the
cortical and subcortical white matter of the occipital lobes bilaterally in a
young woman. There is no evidence of hemorrhage or significant mass effect.
Repeat MR imaging in 2 weeks is normal. This is most consistent with which of the
following?
a. Superior sagittal sinus thrombosis
b. Progressive multifocal leukomalacia
c. Herpetic encephalopathy
d. Eclampsia/Pre-eclampsia
e. Tip of the basilar artery syndrome

Type- Neuro
Answers- d
Notes-

Progressive multifocal leukoencephalopathy (PML) produced by papova viruses is an


uncommon progressive subacute demyelinating disorder of the CNS affecting
immunologically compromised patients, commonly with chronic lymphocytic leukemia
or Hodgkins disease. The diagnosis rests on the histopathologic examination of
brain tissue since laboratory methods are of low yield. The lesions of PML tend to
affect the subcortical white. CT may demonstrate an ill-defined low-density areas.
Usually there is no significant enhancement or mass effect. A differentiation
between PML and lymphoma or leukemic infiltration must be made because they are
commonly associated. The lesions on MRI are seen as high-intensity areas limited
to the white matter. At first the lesions are round or oval, then confluent and
large, with no mass effect. The involvement is most often asymmetric. The findings
are nonspecific; hence, demyelination, infarction, other infections, and malignant
processes should be considered in the differential diagnosis.
The etiology of eclampsia and the other hypertensive encephalopathies is similar.
The brain normally is protected by an autoregulation system that ensures constant
perfusion over a range of systemic pressures. If these autoregulatory limits are
exceeded, passive overdistention of the cerebral arterioles may occur and blood-
brain barrier breakdown ensues. Interstitial extravasation of proteins and fluid
results in multiple foci of reversible vasogenic edema. The posterior circulation
is particularly prone to develop hypertensive encephalopathy related lesions. (D
p857, Osborn p177-8).
Top of the basilar syndrome refers to occlusion of the distal basilar artery. The
predominant lesions are found in the thalami, posterior limb of the internal
capsule, mesencephalon, pons and posterior temporal and occipital lobes (Osborn
p365).
Venous sinus thrombosis (D p275, Osborn p385-95). Herpes Encephalitis (D p230).
1999 Board Question
MR T2 -weighted image (TR 2000 msec, TE 70 msec) of acute sagittal sinus
thrombosis with associated deep venous hemorrhagic infarct. The sagittal sinus is
low in signal intensity because the clot is deoxyhemoglobin (white arrow). The
venous infarction is hemorrhagic. (TF)
by CM

63. Branchial cleft cyst, which is true?

a. Commonly multilocular
b. Commonest at the angle of mandible
c. 1st Branchial arch more common
d. Commonly presents in adolescent

Type- Neuro
Answers- b
Notes-

by KW
Dahnert p301
Commonly in young to middle-aged adults
Second branchial cleft cyst is most common, 95%
They are commonly in the parotid space at the angle of the mandible and
paraphyrangeal space
2000 Board Question

64. Can os odontoidium mimic Type II odontoid fracture?

a. True
b. False

Type- Neuro
Answers- a
Notes-

by KW
1997 Old Board Question

65. Child with dehydration and mental status changes. CT showing bilateral
thalamic hemorrhage with mass effect. Cause:

a. deep vein thrombosis


b. basilar artery thrombosis
c. hemmorhagic PNET

Type- Neuro
Answers- a
Notes-

by KW
Dahnert p274
Dehydration in children is a risk for Sinus Thrombosis
Hemmorhage in gray and white matter unilateral in 2/3, bilateral in 1/3 of cases
Check Osborne for more specific findings
2000 Board Question
Straight sinus/Vein of galen thrombosis with bilateral thalamic infarction

66. Concerning neuro tumors, which relationships are true?

a. Transverse petrous bone fracture - Sensorinueral hearing loss


b. Glomus jugulare - Depressed gag reflex
c. Glomus tympanicum - Pulsitile tinnitus
d. Tentorial meningioma - Nocturnal tinnitus
e. Glomus vagale - Hoarseness

Type- Neuro
Answers- a,b,c,e
Notes-

by KW
Osborn 507,
Jugular foramen - CN IX,X,XI - depressed gag reflex CN IX?
1998 Board Question

67. Concerning neurovascular supply:

a. Anterior choroidal artery supplies the inferior colliculi


b. Anterior inferior cerebellar artery supplies the internal auditory canal
c. Posterior choroidal artery supplies the trigone of the lateral ventricle
d. Superior cerebellar artery supplies the inferior colliculi
Type- Neuro
Answers- b,d
Notes-

The lateral posterior choroidal artery can originate either from P2 or proximal
cortical branches. The lateral posterior choroidal artery courses over the
pulvinar of the thalamus and supplies the posterior thalamus and lateral
ventricular choroid plexus. Osborn140. The anterior choroidal artery usually
arises from the distal internal carotid artery above the posterior communicating
artery origin. The medial and lateral posterior choroidal arteries arise from the
PCA. The three choroidal arteries are in hemodynamic balance with each other and
their vascular territories are complementary. The anterior choroidal artery
typically supplies part of the posterior limb and genu of the internal camsule,
medial globus pallidus, optic tract, temporal lobe uncus, and amygdaloid nucleus
and choroid plexus of the lateral ventricle. osborn363.
The AICA runs toward the cerebellopontine angle and may loop into the internal
auditory canal, where it gives off labyrinthine branches to the inner ear before
supplying the anteroinferior part of the cerebellum. neuro requisites 60.
The superior cerebellar artery is the last infratentorial branch off of the
basilar artery. The lateral marginal and hemispheric branches supply the upper
part of the cerebellum before the vessel terminates in the superior vermian
artery. Other fine branches help supply the pons, the superior cerebellar
peduncle, and the inferior colliculus. The superior vermian vessel anastomoses
with the inferior vermian artery of the PICA. requisites 60.
The anterior choroidal artery supplies (and follows) portions of the optic tract,
medial temporal lobe, uncus, amygdala, hippocampus, anterior limb of the internal
capsule, choroid plexus of the lateral ventricle, inferior globus pallidus,
caudate nucleus, cerebral peduncles, and midbrain. neuro requisites56-57.
1989 Board Question
by KH

68. Concerning Sturge-Weber syndrome:

a. Intracranial calcification
b. Angioma of the retina
c. Intrarenal tumors
d. Rhabdomyoma of the heart
e. Skull thickening

Type- Neuro
Answers- a,e
Notes-

von Hippel Lindau has retinal angiomas


osborn98-100. D268;
1989 Board Question
by KH

69. Concerning the appearance of a trigeminal neuroma:

a. Can be confused with a epidermoid on CT


b. Causes hyperostosis
c. Obliterates meckels cave
d. Can be confused with a meningioma on CT
e. Commonly calcified

Type- Neuro
Answers- c
Notes-

1989 Board Question


by MH

70. Matching concerning the appearance of the corpus callosum:

a. Cavum vergae - seperates posterior columns of fornix


b. Cavum interposition - extension of quadrigeminal plate cistern above 3rd
ventricle
c. Cavum septum pellucidum - separates the frontal horns of lateral
ventricles

Type- Neuro
Answers- a,b,c
Notes-

Cavum septi pellucidi= 5th ventricle; thin triangular membrane consisting of two
glial layers covered laterally with ependyma separating the frontal horns of
lateral ventricles. 80% in term infants/15% in adults; Cavum vergae=6th
ventricle;cavity posterior to columns of fornix; contracts after about 6th
gestational month.; Cavum veli interpositi=extension of quadrigeminal plate
cistern above 3rd ventricle to foramen of Monro, laterally bounded by columns of
fornix + thalums. D188.
1989 Board Question
by KH

71. Concerning the use of contrast material:

a. approximately 30 cc of standard nonionic contrast is the dose for lumbar


myelography
b. approximately 5 cc is the dose for an external carotid artery angiogram
c. approximately 25 cc of standard nonionic contrast is the dose for
cervical myelography
d. approximately 8 cc is the dose for an internal carotid artery angiogram
e. nonionic and ionic contrast material exhibit a similar degree of
nephrotoxicity

Type- Neuro
Answers- b,d
Notes-

1995 ITE True/False Question


Internal carotic artery injection is about 6ml/s for a total of 8cc. External
carotid artery
injection is 3ml/s for 5cc. p 16(Req)
In general, 3.0g limit for iodinated contrast is instilled intrathecally.
Therefore,
10cc of 300mg/ml concentration of 12cc of 240mg/ml concentration is injected for
lumbar or
cervical myelography.--from package insert of Omnipaque. If doing a cervical
myelography via
a lumbar approach, the max dose should be given. p 18
Iodinated contrast agents are potentially nephrotoxic with decreases in creatinine
clearance. The effect on creatinine clearance seems to be less with nonionics.
The potential for
renal damage is greatest in pt with predisposing renal disease/illness.
Ref: Requisites. p16-18
by AE

72. Differential diagnosis of a cerebellopontine angle mass in a child includes:

a. cerebellar astrocytoma
b. facial nerve neuroma
c. PNET
d. epidermoid

Type- Neuro
Answers- a,b,d
Notes-

by KW
Osborn p441, 617
Nonacoustic neuromas cause CPA masses, usually trigeminal neuroma, ? facial nerve
neuroma
Medulloblastomas are usually in the vermis, may be in the lateral cerebellar
hemispheres
Much less than 1% can be from exophytic brainstem/cerebellar astrocytoma
1993, 1988 True/False Question

73. Encephaloceles are associated with which of the following?

a. skull defect
b. microcephaly
c. syndactyly
d. pyelectasis
e. echogenic kidneys

Type- Neuro
Answers- a,b,e
Notes-

Syndactyly= osseous and cutaneous fusion of digits, no assoc. found(it is assoc.


with polydactyly)
Pyelectais= widening of collecting system and ureter, no assoc. found
1993 True/False Question
An encephalocele arises from failure of dorsal induction, which normally occurs at
3-4
weeks gestational age. However, basal (sphenoidal) encephaloceles arise as a
result of abnormal
ventral induction (2 months gestational age). An encephalocele is extracranial
extension of
leptomeninges and brain tissue (meningocele is extracranial extension of
leptomeninges alone).
Encephaloceles are much more common than meningoceles and typically occur along
the
midline.
Nervous system anomalies which are frequently associated with encephaloceles:
Dandy-
Walker malformation and Chiari III malformation (occipital), dysgenesis of the
corpus callosum
(parietal), dysgenesis of the corpus callsum, callosal lipomas and globe anomalies
such as
microphthalmia/anophthalmia (frontoethmoidal), and agenesis of the corpus callsum
and optic
nerve dysplasias (basal).
A skull defect can give rise to a meningoencephalocele - 70% are occipital, 10%
parietal,
and 9% frontal (in the U.S.). Southeast Asian women have a propensity for
nasofrontal or
sphenoethmoidal meningoencephaloceles.
Meckel-Gruber syndrome consists of renal cysts (micro would be echogenic),
microcephaly, polydactyly, and meningoencephaloceles.
Amniotic band syndrome also gives multiple irregular asymmetric off-midline
encephaloceles (among abnormalities found in amniotic band syndrome is distal
syndactyly).
I couldnt find any association with pyelectasis.
References: Lee, Rao, and Zimmerman 1992, pp. 194, 202-204; Dahnert 1993, p. 169,
629
by AE

74. Examples of spinal dysraphism include:

a. myelolipoma
b. sacralization of L5
c. Dandy-Walker cyst
d. diastomatomyelia
e. syringobulbia

Type- Neuro
Answers- d
Notes-

p528
1992 True/False Question
For b. lipoma is associated with a tethered conus which is associated with mild
dysraphism.
The mechanism for development of spinal dyraphism is lack of folding of primodial
neural tissue into a tube. No bone is induced directly over the neural tissue.
Get meningocele,
myelocele, myelomeningocele are tissue that go through a boney spinal defect.
when fat,
meninges, cord roots protrude through the defect its called a
lipomyelomeningocele.
Lipomyelomening is associated with tethered cord and Chiari II and
syringohydromyelia. p
273Req
Dandy Walker is partial or complete absence of the vermis with dilatation of the
fourth
ventricle. p 259
myelolipoma is a benign tumor of the adrenals. p 273 Dunnick
ref: Requisites p 259, 273, Dunnick p 273
by MH

75. Herpes encephalitis:

a. hydrocephalus
b. predilection for temporal lobe
c. normal radionuclide brain scan
d. due to herpes zoster
e. usually fatal

Type- Neuro
Answers- b,e
Notes-

Caused by Herpes simplex I (in adults) and II (in neonates). Mortality 70%.
Brain scintigraphy demonstrates characteristic focal increase in activity in the
temporal lobes. No hydrocephalus (D p230).
by CM
1996 Question

76. Homonymous hemianopsia is most likely to occur from a lesion in which brain
territory?

a. Optic nerve
b. Anterior optic chiasm
c. Posterior optic chiasm
d. Lateral geniculate body
e. Occipital lobe

Type- Neuro
Answers- c,d,e
Notes-

1999 Board Question


by MH

77. In a 4 y/o male a suprasellar mass, the diagnostic possibilities include:

a. craniopharyngioma
b. optic chiasm glioma
c. germinoma
d. Rathke cleft cyst
e. dermoid

Type- Neuro
Answers- a,b,c,d,e
Notes-

1992 True/False Question


The dermoid is unlikely but possible in the suprasellar area.
The germinoma is the most frequent tumor of the hypothalamic area. It is most
commonly pineal but 20% are suprasellar. The peak age is early puberty but it can
be seen in a 4
y/o. M=F with suprasellar germinomas while 90% of pineal germinomas are in males.
The most
common presentation of a suprasellar germinoma is the hypothalamic syndrome
(diabetes
insipidus, emaciation, precocious puberty). They are relatively radio-resistant
but long time
survival rates are high. There is a marked tendency for them to spread through
CSF pathways -
In the suprasellar region, the finding of an infiltrating suprasellar mass that is
intrinsically
hyperdense would favor germinoma over astrocytoma. (Seminars in US, CT, MRI.
Vol.13#6,
Dec. 1992, p. 427)
A hypothalamic hamartoma usually presents before the age of 3 y/o with precocious
puberty. It is isointense to gray matter on all sequences with no enhancement.
It is a rare tumor.
Craniopharyngioma is a common childhood tumor. It accounts for 50% of pediatric
sellar
and suprasellar tumors (90% suprasellar). In children, they tend to be cystic and
enhance. 90%
will be at least partially calcified. They extend into the anterior, middle, or
posterior cranial
fossa in 25% of cases.
Optic glioma is classified as a juvenile pilocytic astrocytoma and represents 2/3
of all
primary optic nerve tumors. The mean age of presenation is 8.5 years ( 5yo in
presence of NF1
and 12yo in abscence of NF). p 291 Requisites
by MH

78. In a patient with acquired hepatocellular degeneration, the basal ganglia are
most likely to demonstrate which of the following signal intensity patterns with
MR?

a. Increased signal on T2
b. Increased signal on T1 and T2
c. Decreased signal on T1
d. Decreased signal on T2
e. Increased signal on T1

Type- Neuro
Answers- e
Notes-

Acquired hepatocerebral degeneration is an irreversible neurodegenerative syndrome


that occurs with many types of chronic liver disease. It is most frequently
associated with alcoholic cirrhosis, subacute or chronic hepatitis, and portal-
systemic shunts. Pathologically, the brain has laminar or pseudolaminar necrosis
with microcavitary changes at the gray-white matter junction, in the corpus
striatum, and in the cerebellar white matter. Imaging changes correlate with
plasma ammonia levels. The typical finding is bilateral basal ganglia
hyperintensities on T1, seen in 50-75% of patients with advanced chronic liver
disease. Other areas of increased signal include the pituitary, caudate,
subthalamic region, and the mesencephalon around the red nucleus. Long-term TPN
may also cause increased T1 in the BG, probably caused by mangenese toxicity.
(Osborn775)
1999 Board Question
by KH

79. In a patient with complete proximal ICA occlusion, an ipsilateral injection is


performed and filling of the distal ipsilateral ICA is seen. What is the supply
of blood to the distal ICA?

a. retrograde flow via the ophthalmic artery


b. from the posterior circulation
c. anterior communicating artery

Type- Neuro
Answers- a
Notes-

by KW
Verified Osborn p143
1991 Single Best Answer Question
The intracranial extracavernous internal carotid artery gives rise to several
important
branches:
a- superior hypophyseal artery
b- ophthalmic artery (anterior falx artery and, occasionally, the middle meningeal
artery arise from it)
c- posterior communicating artery (one or both are hypoplastic in 20% of cases)
d- anterior choroidal artery (divides into proximal (cisternal) and distal
(plexal) segments) - it is an important supply to the choroid plexus of the
temporal horn of the lateral ventricle.
e- small unnamed branches to the hypothalamus, optic tract, and optic chiasm
The ophthalmic artery is an important source of collateral flow in cases of
cervical ICA occlusion.

80. In a patient with multiple cerebral aneurysms, the most reliable finding which
will accurately localize the site of bleeding is:

a. largest aneurysm
b. irregularity of anerysm
c. vasospasm
d. location of the hematoma

Type- Neuro
Answers- a
Notes-

1995 Single Best Answer Question


Clues for which aneurysm is bleeding: largest aneurysm(87%), location of
blood(70%), anterior communicating artery(70%), contralateral side of all
visualized aneurysms(nonvisualization due to spasm)(60%), and irregular contour of
the aneurysm. (Dahnert p216-17) but in Neuro Requisites p138 states, In the case
of multiple aneurysmsthe best indication of the aneurysm that has bled is finding
a hematoma associated with it.
Other findings include:
ACA aneurysm- blood in the interhemispheric fissure, lat. vent
MCA-- blood in the sylvian fissure
PCA or ICA- blood in the 4th vent.
MR is useful to find hemorrhage in the wall of the aneurysm that bled. Angio
findings associated with rupture include: proximity to spasm, irregularity of
aneurysm, tendency for larger aneurysm to bleed, mass effect around aneurysm,
tapering of the apex of aneurysm.
ref: p 138 Neuro Requisites
by CM

81. In ataxia telangiectasia, there is inability to repair chromosomal damage:

a. True
b. False

Type- Neuro
Answers- a
Notes-

1996 Question

82. Infants of diabetic mother can get which congenital abnormality:

a. renal agenesis
b. meningomyelocele
c. caudal regression syndrome

Type- Neuro
Answers- a,c
Notes-

by KW
Dahnert p161,791
Caudal regression associated with diabetic mothers in ~20% of cases
2000 Board Question

83. Infarct of the posterior cerebral artery affects:

a. posterior aspects of temporal and parietal lobes


b. contralateral thalamus
c. ipsilateral thalamus
d. most anterior superior cerebellum
e. frontal horn regions

Type- Neuro
Answers- a,b,c
Notes-

1995, 1992, 1989 True/False Question


The important thing to know is that the posterior cerebral artery supplies the
thalamus via
thalamic perforators. However, there is also cross flow to contralateral as well
as ipsilateral
sides. Therefore, with PCA occlusion you can infarct no thalamus, ipsilateral
thalamus,
contralateral thalamus, or ipsilateral and contralateral thalamus. Most commonly,
the ipsilateral
thalamus is infarcted and only rarely the contralateral thalamus. The PCA does
not supply the
cerebellum. It supplies occipital as well as posterior temporal and parietal
lobes as well as
choroid plexus and ependyma of the third and lateral ventricles. The frontal lobe
is the domain
of the ACA.
Midbrain perforating arteries arise from the basilar and prox PCA. Infarctions of
these
arteries can affect CN III, IV, causing oculomoter deficits; the cerebral
peduncles- affecting
motor strength, medial lemniscus- altering sensation; the red nucleus and
substantia nigra- affects
coordination and motor control,; reticular activation substance- affect level of
consciousness.
The PCA have small premamillary thalamoperforate, and thalamogeniculate branches
supplying the hypothalmus, midbrain, inferior thalamus. Infarctions of these
vessels may affect
memory and emotion- anterior thalamus, endocrine- hypothal, language- pulvinar,
pain
sensation- thalami, sight- lateral geniculate, motor control- subthalamic nuclei.
The next branches of the PCA are the medial and lateral posterior choroidal
arteries that
supply the trigone of the lateral vents. They supply the tectum, choroid plexus
of the 3rd vent,
and the thalami. The lateral posterior choroidal arteries supply the choroid of
the lateral and third
vents and the posterior thalamus , the fornix.( fornix lies anterior and inferior
to the corpus
collosum)
The PCA continues branching off the anterior and posterior temporal ateries and
posterior
pericollosal artey before terminating into the parietooccipital branches and
calcarine arteries.
PCA infarct affects vision- calcarine, memory- post-inferior temporal lobe, smell-
hippocampus,
emotion-splenium.
One named branch off the ACA is the recurrent artery of Heubner which supplies
the
head of the caudate and anteroinferior internal capsule. p 57
ref: Requisites p 61-2
by AE

84. Lesions in which of the following areas can result in homonymous hemianopsia?

a. posterior to the chiasm


b. lateral geniculate
c. anterior to the chiasm
d. in the occipital lobe

Type- Neuro
Answers- a,b,d
Notes-

neuroanatomy atlas207
1988 True/False Question
Neuroradiology Section
by KH

85. Leukenshadel is most commonly associated with?

a. Hydrocephalus
b. Small foramen magnum
c. Dandy Walker
d. Craniosynostosis
e. Myelomenigocele

Type- Neuro
Answers- e
Notes-

by KW
Verified Dahnert p172, Kirks p94
Nearly all patients with Chiari II malformation manifest a dysplasia of the
membranous bones of calvarium called Lckenschdel. This appears as clusters of
thinnings, pits, and fenestrae that are most prominent near the vertex or the
torcula. Lckenschdel may be observed in a fetus in utero as early as at 8 months
of gestation. It typically persists until 1 to 3 months of age and then disappears
after approximately age 6 months, whether or not there is progressive
hydrocephalus. (TF)
1999 Board Question

86. Low lying cerebellar tonsils (Chiari I malformation) is most closely


associated with?

a. Prominent massa intermedia


b. Small posterior fossa
c. Meningomyelocele
d. Beaking of the tectum
e. Syringomyelia

Type- Neuro
Answers- e
Notes-

Beaking of the tectum is seen with Chiari II, Luckenschadel skull also with Chiari
II
Chiari described four unrelated malformations of the posterior fossa, now called
Chiari I, II, III, and IV. In simplified form, the Chiari I malformation
consists of protrusion of the cerebellar tonsils downward through the foramen
magnum; obliteration of the cisterna magna; and variable, usually very mild,
elongation and caudal displacement of the fourth ventricle, brain stem, and
cerebellum. Chiari I malformation is associated with hydrosyringomyelia (60%-70%
of patients), hydrocephalus (20%-25%), and segmentation anomalies of the
craniovertebral junction including basilar impression (25%), assimilation of C1 to
the occiput (10%), Klippel-Feil (10%), and incomplete ossification of the C1 ring
(5%). It is not associated with myelomeningocele.
Many patients with Dandy-Walker malformation show persistent fetal position of
tentorium and sinuses, so the tentorium lies in a nearly vertical position along
the parietal bones and the torcular lies far above the lambda ( torcular-lambdoid
inversion). The posterior tentorium may be dehiscent with terminal duplication of
the straight sinus. The incisura in the tentorium is inclined nearly vertically
with the tentorial leaves and is often wider than normal. In axial CT sections,
the tentorial leaves and the incisura usually form an inverted-V
The Chiari II malformation is the most common serious malformation of the
posterior fossa. It is a complex deformity of the calvarium, dura, and hindbrain,
which nearly always is associated with spina bifida and (meningo) myelocele. In
aggregate, the changes create a bony and dural posterior fossa that is too small
for the brain stem and cerebellum. Then, as the brain grows, it erodes the walls
of the posterior fossa and extrudes into the spinal canal below and the
supratentorial space above. In this process, the posterior fossa contents mold and
become molded by the structures upon which they impinge. The changes induced in
the very fluid immature brain become set by the process of myelination and appear
to persist relatively unchanged despite later alterations in the bone. The
midbrain is abnormal in nearly every case. Characteristically, the four colliculi
are at least partially fused into a conical mass designated the beak. Since the
beaking affects the inferior colliculi more than the superior colliculi, beaking
is seen best on low CT sections caudal to the expected position of midbrain. The
upper midbrain may, in fact, appear deceptively normal.
1999 Board Question
by AE

87. Match the following neuro anatomy:

a. Dilantin therapy - Hippocampus


b. Huntingtons disease - Caudate nucleus
c. Wernickes encephalopathy - Mamillary bodies
d. Mesial temporal sclerosis - Hippocampus

Type- Neuro
Answers- b,c,d
Notes-

per MH
1998 Board Question
by MH

88. Matching regarding calvarium:

a. Microcrania - Trisomy 13
b. Macrocrania - Chiari 1
c. Sagittal suture synostosis - Scaphocephaly
d. Brachycephaly - Aperts syndrome

Type- Neuro
Answers- a,c,d
Notes-

by KW
1997 Old Board Question
Brachycephaly - closure of the coronal and lamboid sutures

89. Matching regarding causes of the following neuro conditions:

a. Fatty acid metabolism - Adrenoleukodystrophy


b. Post viral - ADEM
c. Electrolyte correction - Pontine myelinolysis

Type- Neuro
Answers- a,b,c
Notes-

by KW
Osborne p704,724
Acute Disseminated Encephalomyelitis = Immune mediated post viral demyelination
Mostly in children and young adults
Subcortical hyperintensity foci on T2
Adrenoleukodystrophy = X linked deficiency of single peroxisome enzyme involved in
oxidation of fatty acids
1997 Old Board Question

90. Matching regarding Chiari:

a. Inversion of the tentorium - Chiari I


b. Widened occipital horns - Chiari II
c. Frontal horns pointing inferiorly - Chiari II
d. Syringohydromyelia - Chiari I and Chiari II

Type- Neuro
Answers- b,c,d
Notes-

by KW
Dahnert p223
Chiari I = Adult, Chiari II = Childhood
Chiari I and II are associated with syringohydromyelia (20-30%)
Chiari II - also associated with syringhydromyelia
Bat wing configuration of frontal horn - horns point inferiorly
There is elevation of the tentorium with Chiari II
There is colpocephaly, or enlargement of occipital horns
1997 Old Board Question

91. Matching regarding phakomatosis:

a. Hemangioblastoma - von Hippel Lindau


b. Venous angioma - Sturge Weber
c. Giant cell astrocytoma - Tuberous sclerosis

Type- Neuro
Answers- a,b,c
Notes-

by KW
Dahnert p268, 272, 275
1997 Old Board Question

92. MR findings in a 12 year old reveal a prominent optic chiasm and a focus of
increased T2 signal within the basal ganglia and cerebellum. This most likely
represents?

a. Neurofibromatosis 2
b. Von Hippel Lindau
c. Sturge Weber
d. Tuberous Sclerosis
e. Neurofibromatosis 1

Type- Neuro
Answers- e
Notes-

Multiple Hamartomas - "Unidentified Bright Objects" in basal ganglia, brainstem,


cerebellum
by KW
Osborne p81
1999 Board Question

93. On an injection of contrast into the left vertebral artery, there is non-
visualization of the ipsilateral posterior cerebral artery. Which of the
following are possible causes?

a. occlusion of the left posterior cerebral artery


b. spasm
c. aplasia
d. origin from the left internal carotid artery
e. flow phenomenon

Type- Neuro
Answers- a,b,d,e
Notes-

1994, 1992, 1989v True/False Question


Carotid-basilar anastomoses are persistent embyological anastomoses.
Possible carotid ---> basilar anastomoses:
1) persistent trigeminal artery (most frequent, discovered in 0.1-0.2% of cerebral
angiograms) -
arises just before the cavernous segment of the ICA.
2) otic artery
3) hypoglossal artery
4) suboccipital artery
5) proatlantal intersegmental artery
6) posterior communicating artery
by MH

94. On CT myelograms, filling of the optic nerve sheath represents:

a. optic nerve glioma


b. increased intracranial pressure
c. normal variant
d. optic sheath meningioma

Type- Neuro
Answers- c
Notes-

by KW
Dahnert p283 lists Patulous subarachnoid space as a cause of optic nerve
enlargement
Per Dr. Acker
1992 Single Best Answer Question

95. On SPECT imaging (ECD), which pattern does patient with Alzheimer have:

a. increased activity in frontal lobe


b. decreased activity in the parietal lobe
c. increased activity in the temporal lobe

Type- Neuro
Answers- b
Notes-

by KW
Per Tim Baker
Decreased uptake in the temporoparietal region
Check source
2000 Board Question

96. Patient with neurofibromatosis and has pulsatile exophthalmos:

a. Meningioma
b. Enlargement of the optic foramina
c. Sphenoid wing dysplasia
d. CC fistula

Type- Neuro
Answers- c
Notes-

by KW
Dahnert p261
Pulsitile exopthalmos due to herniation of subarachnoid space and temporal lobe
into orbit...
Could be enlargement of the optic foramina or Sphenoid wing dysplasia...
But p260 states that defect in sphenoid bone leads to extension of middle cranial
fossa into orbit
Also states that there is concentric enlargement of optic foramen due to optic
glioma
2000 Board Question
Patients with neurofibromatosis may be pulsatile exophthalmos due to absence of
the greater wing of sphenoid bone. The pulsation results from transmitted cranial
pulsation

97. Patient with small subcortical shear injury. Which is the best MR sequence for
evaluating it:

a. Flair sequence
b. T1, T2 spin echo
c. Gradient echo T2

Type- Neuro
Answers- c
Notes-

by KW
2000 Board Question

98. Patient with wernicks encephalopathy would have atrophy of which cranial
structure:

a. caudate lobe
b. hippocampus
c. mamillary body
d. substantia niagra

Type- Neuro
Answers- c
Notes-

Osborn p764
by KW
2000 Board Question

99. Periventricular parenchymal calcification is seen in:

a. neurofibromatosis
b. Sturge-Weber
c. TORCH
d. tuberous sclerosis

Type- Neuro
Answers- c,d
Notes-

Sturge-Weber- capillary venous angiomas of face, choroid of eye, leptomeninges;


tram-track gyriform cortical calcifications
Tuberuos sclerosis- subependymal hamartomas along lateral vents that calcify with
increasing age
Toxoplasmosis/CMV- periventricular calcifications
1991 True/False Question
Sturge-Weber calcification occurs in the parenchyma. In TORCH, toxoplasmosis and
CMV do it.
by AE

100. Pineal tumors include:


a. germinoma
b. PNET
c. teratoma

Type- Neuro
Answers- a,b,c
Notes-

1994 True/False Question


Pineal region masses include pineoblastoma which is a PNET, germinoma, teratoma.
Also
included are pineocytoma, chorioca, embryonal cell, glioma.
ref: p 95 , 83 Requisites
by AE

101. Possible causes of pulsatile tinnitus include:

a. anterior communicating artery aneurysm


b. glomus jugulare tumor
c. aberrant carotid with carotid canal dehissence
d. glomus tympanicum tumor
e. cholesteatoma

Type- Neuro
Answers- b,c,d,e
Notes-

,?e(aneurysm is listed as a cause, but unsure about ant comm artery)


Req p.345
1992 True/False Question
The differential diagnosis of pulsatile tinnitus:
paraganglioma (including glomus tympanicum, which arises at the cochlear
promontory of the middle ear cavity, and glomus jugulare)
dural AVM
aberrant internal carotid artery
dehiscent or high jugular bulb
high-grade carotid artery stenosis
petrous internal carotid artery aneurysm or dissection
fistula of temporal bone region (cavernous carotid)
cholesteatoma, cholesterol granuloma
meningioma
Menieres disease
Reference: Grossman & Yousem 1994, pp. 343-348
by AE

102. Punctate hemorrhages near the vertex are most likely due to:

a. Superior sagittal sinus thrombosis


b. Amyloidosis

Type- Neuro
Answers- b
Notes-

by KW
1997 Old Board Question
103. Regarding anatomy of the orbit:

a. annulus of zinn divides the globe into anterior and posterior compartment
b. the inferior oblique is supplied by the trochlear nerve
c. the trochlea is situated on the lateral side of the orbit
d. the ophthalmic artery is situated inferior to the superior opthalmic vein

Type- Neuro
Answers- d
Notes-

by KW
See figure on Dahnert p285
Lateral Rectus - Abducens nerve
Superior Rectus Oblique - Trochlear nerve
2000 Board Question

104. Regarding anencephaly:

a. oligohydramnios is seen early in the 2nd trimester


b. most common in males
c. brainstem and some mesencephalon structures are present
d. frequently associated with other major organ system abnormalities

Type- Neuro
Answers- c
Notes-

M:F=1:4;Polyhydramnios (40-50%) after 26 weeks GA (due to failure of normal fetal


swallowing/ oligohydramnios; other cns/ spinal obnormalities;
hypophysis+rhombencephalic structures usually preserved. D215.
1991 True/False Question
Anencephaly is associated with spinal dysraphism and polyhydramnios. p 250 Req
Spinal and non CNS abnormalities and polyhydramnios is common. Anencephaly is
characterized by absence of the cranial vault, cerebral hemispheres, diencephalic
structures and their replacement by amorphous vascular-neural mass. p 883-4 Rumack
by KH

105. Regarding carotid-vertebral connections?

a. Tentorial artery
b. Recurrent artery of Huebner
c. Otic artery
d. Occipital artery

Type- Neuro
Answers- c,d
Notes-

Req p.55, ECA anastomose to vertebral via occipital, ascending pharyngeal, and
posterior auricular arteries.
Several fetal anastomotic channels may persist rarely--> trigeminal a., otic a.,
hypoglossal a., proatlantal a.
Recurrent artery of Huebner is a lenticulostriate branch of the ant. cerebral
artery, supplies the anterioinferior
portion of the caudate head, putamen, anterior limb of the internal capsule.
1998 Board Question
The authors describe the case of a 67-year-old man with acquired obstruction of
the common carotid artery and a large, direct occipital-vertebral artery
anastomosis bridging the vertebral and carotid artery systems. They discuss the
hemodynamics within this type of anastomosis in the context of cerebrovascular
occlusive disease and outline their classification of the hemodynamic patterns
that have been observed. Surgical procedures for occlusive disease in the presence
of such anastomoses are also addressed. Surg Neurol 1989 Nov;32(5):350-5
by AE

106. Regarding cavum septum pellucidum:

a. indicates hydrocephalus
b. pathologic
c. none of the above
d. rare in newborns

Type- Neuro
Answers- c
Notes-

1994 True/False Question


The frontal horns and the body of the two lateral ventricles are separated by the
septum pellucidum. The two layers containing CSF are consistently seen in
infants. In the majority, they fuse but if they dont it is a cavum septum.
Posterior extention is the cavum verge. The septum pellucidum has neural
connections with the cerebral hemispheres.
p 103 Lee, Rao, Zimmerman. All the answers are false (D p188).
by CM

107. Regarding CT and MR appearance of Multiple Sclerosis:

a. MRI can show plaques in the optic nerves in multiple sclerosis.


b. A CT scan with contrast shows plaques from multiple sclerosis in the
white matter.
c. T2 weighted MRI shows high intensity lesions in the white matter in
multiple sclerosis.
d. A CT scan without contrast material shows plaques from multiple sclerosis
in the white matter.
e. T1 weighted images on MRI show high intensity signal in the white matter
in multiple sclerosis.

Type- Neuro
Answers- a,b,c,d
Notes-

by KW
Verified Osborn 757,761
1989 Board Question

108. Regarding gliomas:

a. pilocytic neoplasms are typically lobular, well-marginated, and not


calcified
b. 80% of low grade lesions eventually degenerate into high grade lesions
c. the most common presenting age for gliomas is approximately 50 years old
d. there is a poor prognosis associated with pilocytic astrocytoma with 10-
20% ten year survival
e. 70-80% of gliomas in children originate in the brain stem
Type- Neuro
Answers- c
Notes-

1994 True/False Question


Less than 70-80% of childhood gliomas are from the brain stem. Junvenile
pilocytic astrocytomas are the most common infratentorial neoplasm in the juvenile
age group. p 83 Neuro Requisites (D p234-5).
50% of benign astrocytomas will degenerate into anaplastic astrocytomas &
glioblastoma(Osborn p532).
Pilocytic astrocytomas are indolent, slowly growing neoplasms with a 10 year
survival of 75%(Osborn p555).
Pilocytic astrocytomas typically are round or oval, sharply demarcated and
smoothly marginated masses. Calcification occurs in 10%(Osborn p555).
Although the peak incidence of gliomas is listed as the seventh decade in Osborn
p407; in Dahnert p234, age peak is listed as middle adult life. Similar age
peaks(about 50 years or older) are given in the Primer p486.
by CM

109. Regarding idiopathic orbital pseudotumor:

a. primary therapy is radiation therapy


b. the cause is often unknown
c. often involves the lacrimal gland
d. can extend intracranially
e. insiduous onset

Type- Neuro
Answers- b,c
Notes-

Primary treatment is steroids, Diff: lymphoma, thyroid ophthalmopathy


1995 True/False Question
Orbital pseudotumor is a common cause of unilateral exophtalomos. Clinical
features
include restriction of ocular motility, chemosis, lid swelling, pain. They
usually have rapid onset
and repond to steroids. There is a chronic progressive from which responds poorly
to steroids.
and is treated with chemo and XRT. In the acute case, it is a inflammatory
condition that may be
an autoimmune condition involving the lacrimal gland, extraocular m, connective
tissue
surrounding the dura of the optic nerve, the orbital fat, epibulbar connective
tissue, and sclera.
May present as a lacrimal mass.
Tolosa-Hunt syndrome is a inflammatory condition similar to orbital pweudotumor
that
involves the cavernous sinus and orbital apex. This presents with painful
opthalmoplegia. p
297
ref: Requisites
true-- may be autoimmune/ assoc. with other autoimmune diseases
by AE

110. Regarding intraventricular meningioma:

a. spreads along CSF pathways


b. most common location is the atrium

Type- Neuro
Answers- b
Notes-

most common radiation-induced CNS tumor(latent period of 19-35 years), uncommon in


children- if seen consider NF2
Atrium/Trigone of the lateral vent is the most common location for an
intraventricular meningioma
1991 True/False Question
Most common location for a meningioma is supratentorial. In descending order, the
common
locations include : parasagittal dura, convexities, sphenoid wing, CPA, olfactory
groove, planum
sphenoidale. p 68 Req
by AE

111. Regarding meningiomas:

a. located in the atria of the lateral ventricle


b. associated with overproduction of CSF
c. uncommon in the spine in children
d. bright on T2

Type- Neuro
Answers- a,c,d
Notes-

by KW
Osborn 587,900
1993 True/False Question
Meningiomas are hypo to iso on T1 to gray matter and iso to hyperintense on T2 to
gray
matter. Intraventricular meningiomas typically occur around the choroid in the
trigone of the
lateral venticle esp the left lat vent. p 68-73 Requisites

112. Regarding Neurofibromatosis Type I:

a. macrocrania
b. sphenoid wing dysplasia
c. optic nerve glioma
d. acoustic neuroma

Type- Neuro
Answers- a,b,c
Notes-

by KW
Verified Osborn 73,82
1995 True/False Question
NF-1 is von Recklinghausens disease. To dx must have two or more of the following
findings-- >6 cafe au lait spots, >2 lisch nodules of the iris, >one plexiform
meurofibroma,
axillary freckling, >one bony dysplasia, pseudarthrosis of long bone, optic
glioma, 1st degree
relative with NF. Acoutic neuroma is associated with NF-2.
ref: Requisites p 266
The orbit frequently displays a characteristic unilateral defect fo the greater
and lesser
wings of the sphenoid attributed to underlying mesodermal dysplasia. Other cranial
abnomalities
include bone defect along left side of the lambdoid suture, hypoplastic maxillary
and ethmoid
sinuses. Themandible and maxilla may be affected by overlying soft tissue
abnormalities. p
1220 Resnick.
Macrocranium is seen in 75%. Manaster p 326

113. Regarding orbital anatomy, which is true?

a. The annulus of Zinn divides the globe into anterior and posterior
chambers
b. The trochlea for the superior oblique is on the lateral wall of the orbit
c. The superior ophthalmic artery is superior to the vein
d. The trochlear nerve innervates the superior oblique muscle

Type- Neuro
Answers- d
Notes-

The extraocular muscles along with the intermuscular membrane divide the
retrobulbar space into the intraconal and extraconal compartments. Six extrinsic
muscles coordinate the movement of the globe: the superior, inferior, medial, and
lateral recti and the superior and inferior oblique. The four rectus muscles arise
at the orbital apex from the anulus of Zinn, a tendinous band encircling the
orbital end of the optic canal and the medial portion of the superior orbital
fissure. (TF) The iris divides the anterior and posterior chambers.
a.(Head & Neck Imaging by Som p751) b.(Som p754) c.(Netter Plate 80) d.(Netter
Plate 79)
1998 Board Question
by CM

114. Regarding orbital pseudotumor:

a. responds rapidly to steroid and radiation therapy


b. well-defined optic nerve on CT
c. muscle insertion spared
d. painless
e. no enhancement on CT

Type- Neuro
Answers- a
Notes-

Dahnert p292
1993 Single Best Answer Question
Grave disease may result in thyroid ophthalmopathy, which is the most common cause
of
bilateral exophthalmos in an adult. Pathologically, there is deposition of
mucopolysaccharides
and infiltration by lymphocytes in the muscle bodies and retroorbital fat.
Imaging reveals
bilateral enlargement of the muscle bodies, with sparing of their tendinous muscle
insertions on
the globe. This is in contrast to orbital pseudotumor (myositis), wherein the
tendinous insertions
are involved
Also see a True/False question in this section on the same topic
Orbital pseudotumor is a common cause of unilateral exophtalomos. Clinical
features
include restriction of ocular motility, chemosis, lid swelling, pain. They
usually have rapid onset
and repond to steroids. There is a chronic progressive from which responds poorly
to steroids.
and is treated with chemo and XRT. In the acute case, it is a inflammatory
condition that may be
an autoimmune condition involving the lacrimal gland, extraocular m, connective
tissue
surrounding the dura of the optic nerve, the orbital fat, epibulbar connective
tissue, and sclera.
May present as a lacrimal mass.
Tolosa-Hunt syndrome is a inflammatory condition similar to orbital pweudotumor
that
involves the cavernous sinus and orbital apex. This presents with painful
opthalmoplegia. p
297
ref: Requisites
by MH

115. Regarding pituitary microadenoma, which of the following are true?

a. prolactin-secreting adenomas are the most common hormonally active


adenomas
b. extension into the cavernous sinus causes extensive amounts of prolactin
secretion/excretion
c. at presentation most are <1 cm in diameter
d. bromocryptine treated patients commonly hemorrhage in the pituitary fossa
e. usually centrally located in the pituitary

Type- Neuro
Answers- a,b,c,d
Notes-

Serum prolactin levels of more than 200ng/ml are highly specific for prolactin-
secreting adenomas whereas markedly elevated prolatin levels(>1000 ng/ml) imply
cavernous sinus invasion. neurorequisties313.
1994 True/False Question
From Osborne, page 467
By definition all microadenomas are less than 1 cm
There are many microadenomas discovered as incidental findings at autopsy
therefore most are not prolactin-secreting.
Pituitary adenomas are the most common intrasellar tumor, representing 10-15% of
all intracranial neoplasms. Prolactinomas account for 40-50% of hormonally active
adenomas; less commonly, adenomas may produce growth hormone (20%) or ACTH (20%).
T1-weighted images are best for visualizing the pituitary because the bright
signal of the surrounding CSF on T2-weighted images obscures the gland.
Hemorrhage into macroadenomas is not uncommon. Reference: Atlas, pp. 632-640
Treatment with bromocriptine increases the likelihood of hemorrhage which has been
reported in more than half of the bromocriptine treated adenomas. ? Hemmorrhage
into the gland not into the fossa. p 315 Requisites
by KH
116. Regarding spinal meningiomas:

a. intradural extramedullary
b. more common in cervicothoracic region
c. more common in women
d. hyperintense on T2

Type- Neuro
Answers- a,c
Notes-

Most intracranial lesions can be bright on T2, but spinal meningiomas are usually
isointense
Most are typical benign meningiomas; second most common cause of spinal tumor;
classic patient is a middle-aged woman; most common location; thoracic spine; 90%
are intradural extramedullary; Imaging findings-Bone erosion, CA++ rare; most are
isointense with cord on T1 and T2WI; moderate contrast enhancement; +/- dural tail
Osborn899
1991 True/False Question
by KH

117. Regarding the appearance of Sturge-Weber syndrome:

a. intracranial calcification
b. intrarenal tumors
c. skull thickening
d. rhabdomyoma of the heart
e. angioma of the retina

Type- Neuro
Answers- a,c
Notes-

1989 True/False Question


Sturge-Weber syndrome is also known as encephalotrigeminal angiomatosis. Findings
include port wine nevus of the face (V1 of the trigeminal nerve), mental
retardation, seizures, leptomeningeal angiomatosis, and glaucoma, also
choroidal/scleral hemangiomas. Gyriform or tram-track pattern of calcification is
the most common. It typically occurs in the parieto-occipital region. The
calcifications are deposited in gyri beneath venous angiomas and in the meninges.
The involved hemisphere is usually atrophic leading to ipsilateral calvarial
thickening. Angiomas of the retina are found in Von Hippel-Lindau.
Rhabdomyoma of the heart is found in tuberous sclerosis. By intrarenal tumors
they probably mean angiomyolipoma, which is also found in TS. Renal angiomatous
malformations are possible. Wyburn-Mason syndrome may be forme fruste of S-W--
have facial vascular nevus in nerve V, retinal angioma, and midbrain AVM. p 271
Req. (D p268-9)
by CM

118. Regarding the appearance of the basal ganglia in chronic liver disease on
MRI?

a. Low signal on T2
b. High signal on T1
c. High signal on T2
d. Paramagnetic effects on T2
e. Low signal on T1
Type- Neuro
Answers- b
Notes-

Requisites p236
B.Acquired hepatocerebral degeneration is an irreversible neurodegenerative
syndrome that occurs with many types of chronic liver disease. It is most
frequently associated with alcoholic cirrhosis, subacute or chronic hepatitis, and
portal-systemic shunts. Pathologically, the brain has laminar or pseudolaminar
necrosis with microcavitary changes at the gray-white matter junction, in the
corpus striatum, and in the cerebellar white matter. Imaging changes correlate
with plasma ammonia levels. The typical finding is bilateral basal ganglia
hyperintensities on T1, seen in 50-75% of patients with advanced chronic liver
disease. Other areaas of increased signal include the pituitary, caudate,
subthalamic region, and the mesencephalon around the red nucleus. Long-term TPN
may also cause increased T1 in the BG, probably caused by mangenese toxicity.
(O775)
1998 Board Question
by MH

119. Regarding the signs of a carotid-cavernous fistula:

a. Marked filling of the superior sagittal sinus


b. Prominent Vein of Galen
c. Retrograde filling of the ipsilateral superior ophthalmic vein
d. Reversal of flow in the ipsilateral ophthalmic artery

Type- Neuro
Answers- c
Notes-

Dahnert p286
Evaluation of carotid-cavernous sinus fistula should be assessed by bilateral
selective arteriography of the internal and external carotid arteries. Traumatic
carotid-cavernous sinus fistula is usually caused by a fracture through the
cavernous sinus; few cases are caused by surgical trauma. The spontaneous type is
usually caused by a dural malformation about the cavernous sinus. They are often
found in elderly women who suddenly develop proptosis, blurred vision, and dilated
conjunctival vessels. All these cases have a greatly dilated superior ophthalmic
vein on the side of proptosis. (TF)
For the investigation of a carotid cavernous fistula, contrast CT, MRI, and
conventional angiography are utilized for determination of the size of the
fistula, along with demonstration of associated dilated venous channels,
especially the superior ophthalmic vein. CT, following bolus injection of
iodinated contrast material, demonstrates a dilated cavernous sinus and enlarged
and tortuous superior ophthalmic vein. (TF)
Route of drainage from a carotid-cavernous fistula is through the superior
ophthalmic vein (common), the contralateral cavernous sinus, the petrosal sinus,
and rarely the cortical veins. Patients present with pulsating exopthalmos,
chemosis, conjunctival edema, and a persistant bruit. There is a decrease in
extraocular movement and in vision due to increased intraocular pressure (50%)
which is an indication for immediate treatment. (P219)
1998 Board Question
by MH

120. Schizencephaly represents which type of developmental abnormality?

a. In-utero infection
b. Migrational abnormality
c. Neural tube closure defect
d. Induction defect

Type- Neuro
Answers- b
Notes-

by KW
Verified Dahnert p267
Occurs 30-60 days gestation
Schizencephaly is a segmental developmental failure of cell migration to form
cerebral cortex, or destruction of a portion of the germinal matrix, occurring
before the formation of the hemispheres. Associated with polymicregyria,
microcephaly, and gray matter heterotopia. Types: clefts with fused lips (walls
oppose one another obliterating the CSF space), clefts with separated lips (CSF
fills cleft from lateral ventricle to subarachnoid space). Located mostly near
the pre- and post-central gyri; uni- or bilateral. Pathognomonic: gray matter
lining cleft extending through entire hemisphere.
1999 Board Question

121. Sources of epidural hematoma include:

a. middle meningeal artery


b. vein of Galen
c. superficial temporal artery
d. superior saggital sinus
e. diploic veins

Type- Neuro
Answers- a,d,e
Notes-

Dahnert p232
1992 True/False Question
10% of epidurals are venous
An epidural hematoma is collection of blood along the subperiosteal inner table.
Itis
associated with skull fracture in 90%. It is biconvex, and since it is
subperiosteal, it is limited by
the sutures (exception is when a fracture crosses a suture). Sources of blood are
the meningeal
vessels (arteries (60%) or veins), dural sinuses, diploic veins, and marrow
sinusoids. The
meningeal vessels are displaced from the inner table. ref: Req p 151
Collection of blood along the subperiosteal outer table is called a
cephalohematoma. An
epidural hematoma can decompress through a fracture to cause a cephalohematoma or
subgaleal
hematoma.
Subdural hemorrhages are due to tears of bridging veins. There is dissection of
the
subdural space with dural vessels remaining close to the inner table. SDH may be
due to minor
trauma or no trauma with such things such as bleeding diatheses. A SDH is usually
crescentic
and, since it is not subperiosteal, crosses regions of sutures.
by MH
122. Structures within the superior orbital fissure include:

a. ophthalmic artery
b. optic nerve
c. maxillary portion of the trigeminal nerve
d. superior ophthalmic vein
e. oculomotor nerve

Type- Neuro
Answers- d,e
Notes-

1992 True/False Question


The superior orbital fissure contains cranial nerves III (oculomotor), IV
(trochlear), V1 (ophthalmic branch of trigeminal), and VI (abducens) as well as
the superior ophthalmic vein, sympathetic fibers, and the orbital branch of the
middle meningeal artery. The ophthalmic artery goes through the optic canal on
the undersurface of the optic nerve. The maxillary portion of the trigeminal
nerve (V1) goes through the inferior orbital fissure. (Osborn p487)
by CM

123. Submental lymph node is classified as what level node:

a. 1
b. 3
c. 2

Type- Neuro
Answers- a
Notes-

by KW
Som p564
I - submental and submandibular
II - Suprahyoid IJ chain
III - Infrahyoid IJ chain to level of omohyoid
IV - Infraomohyoid IJ chain
V - Posterior triangle
VI - Thyroid nodes
VII - Tracheoesophageal nodes
2000 Board Question

124. Symptoms associated with L4 radiculopathy include:

a. absent knee jerk reflex


b. hip pain radiating to the lateral thigh
c. hamstring weakness
d. gluteal pain
e. absent ankle jerk reflex

Type- Neuro
Answers- a
Notes-

1994 True/False Question


L4 radiculopathy should present as hip pain radiating to the anterior thigh (not
lateral), knee, and medial calf. Sensation is impaired over the medial calf and
the territory of the saphenous nerve. One gets an absent knee jerk although
overlapping innervation may make it
difficult to detect. Muscle weakness is difficult to establish owing to the
heavy overlap of root innervation. The quadriceps, abductor longus, gluteus
medius, and tibialis anterior muscles may be weak.
by KH

125. The anterior longitudinal ligament is disrupted in which fracture?

a. pillar fracture
b. hyperextension strain
c. hyperextension dislocation

Type- Neuro
Answers- c
Notes-

by KW
Dahnert p169
Hyperextension teardrop fracture = avulsion of anteroinferior corner by anterior
ligament (longitudinal?)
2000 Board Question
Whiplash or Hyperextension Strain
Whiplash or hyperextension strain is manifested on the x-ray by kyphotic angle of
greater than 10 degrees and a fanning of interspinous process of 12 mm or more. On
MRI examination whiplash injuries may show separation of the disc from the
vertebral end-plate, anterior longitudinal ligament injuries, anterior annular
tear, occult anterior vertebral end-plate fracture.
Hyperextension Dislocation
Hyperextension injuries have two mechanisms, direct anterior craniofacial trauma
and rear-end motor vehicle collision resulting in forceful extension of the head
(whiplash). It may be associated with disruption of the anterior longitudinal
ligament and rupture of the adjacent disc. This injury is associated with
significant morbidity and mortality due to the spinal cord damage. Older patients
with spondylosis are more likely to have this injury because of the limited
elasticity of the soft tissues. The majority of this injury appears in the neck
and lower cervical spine. Radiographically, the changes are subtle, wide disc
space, small avulsion fracture of the anterior-inferior end-plate, prevertebral
soft tissue swelling is seen. MRI shows disruption of the anterior longitudinal
ligament, disc disruption, prevertebral fluid and cord injury. This is an unstable
and significant trauma.

126. The associated findings in diastematomyelia are:

a. scoliosis
b. widened pedicles
c. bone spicule
d. associated lymphoma
e. occult spinal dysraphism

Type- Neuro
Answers- a,b,c,e
Notes-

Dahnert p.165,Diastematomyelia= Split cord secondary to split notochord; also


assoc. with clubfoot, myelomeningocele,
tethered cord, vertebral anomalies
by AE
1996 Question

127. The cause of the wallenburg syndrome is:

a. occlusion of ipsilateral PICA


b. occlusion of ICA
c. occlusion of superior cerebellar artery
d. occlusion of contralateral PICA

Type- Neuro
Answers- d
Notes-

Ipsi face, Contra body findings


by KW
Osborne p368
2000 Board Question

128. The following are phakomatoses:

a. ataxia telangiectasia
b. tuberous sclerosis
c. neurofibromatosis
d. Gardners syndrome
e. Peutz-Jeghers

Type- Neuro
Answers- a,b,c
Notes-

also Von Hippel-Lindau, Sturge-Weber, Osler-Weber-Rendu, Klippel-Trenaunay, Gorlin


1991, 1988 True/False Question
Von Hippel-Lindau disease is an autosomal dominant disorder characterized by
retinal
angiomas, cerebellar and spinal cord hemangioblastomas, angiomas of the liver and
kidney,
pheochromocytomas, and cysts of the pancreas, kidney, and liver. Renal cell
carcinomas also
arise more frequently in these patients.
Sturge-Weber disease is a congenital disorder characterized by angiomatosis
involving
the face, the choroid of the eye, and the leptomeninges. Calcifications occur in
the cerebral
cortex underlying the meningeal angiomatosis and are the most frequent CT finding.
The
calcifications are gyriform in shape, and are most frequently seen in the
temporoparietooccipital
regions.
Tuberous sclerosis is characterized by multiple subependymal hamartomas Giant
cell
astrocytoma refers to enlarging subependymal nodules that tend to be situated near
the foramina
of Monro; they are seen in 5-10% of patients with tuberous sclerosis.
Phakomatoses refer to a group of hereditary diseases of the neuroectoderm
characterized
by cutaneous manifestations. Included are NF, TS, VHL, Sturge Weber. Also
included in the
phakomatosis is meningiomatosis which is hamartomatous proliferation of meningeal
cells via
the intraparenchymal blood vessels in the cerebral cortex AND neurocutaneous
melanosis which
is characterized by cutaneous nevi and melanotic thickening. p 266-72
by AE

129. The most common cerebral brain tumor in a child is:

a. Hemangioma
b. Meningioma
c. Glioma

Type- Neuro
Answers- c
Notes-

by KW
Dahnert p191
Gliomas are 50% of intracranial tumors
Medulloblastoma is 15% of intracranial tumors
Hemangioma - not a cerbral brain tumor
1997 Old Board Question

130. The most common location of an intraventricular meningioma is:

a. body of the lateral ventricle


b. atria of the lateral ventricle
c. foramen of Monro
d. posterior third ventricle
e. fourth ventricle

Type- Neuro
Answers- b
Notes-

by KW
Osborn p588
1992 Single Best Answer Question
Intraventricular meningiomas occur around chorid plexus in the trigone of the
lateral
ventricle (left > right). 15% are in the 3rd ventricle and 5% are in the 4th
ventricle. While
associated with neurofibromatosis (type II), most patients with intraventricular
meningiomas
dont have neurofibromatosis. (Grossman and Yousem 1994, p. 70)

131. The most common posterior fossa mass in a child:

a. Glioma
b. Choroid plexus papilloma
c. Hemangioblastoma
d. PNET
e. Ependymoma

Type- Neuro
Answers- a
Notes-
by KW
Dahnert p249 - We think that Dahnert is wrong.... Probably Glioma/Cerebellar
Astrocytoma
Also most malignant infratentorial neoplasm
But only 15% of pediatric brain tumors... Glioma most common overall
1997 Old Board Question

132. The most common primary cerebellar tumor in a child is:

a. hemangioblastoma
b. medulloblastoma
c. ependymoma
d. cerebellar astrocytoma

Type- Neuro
Answers- d
Notes-

Cerebellar astrocytomas are the most common posterior fossa tumor in this age
group (in some series medulloblastoma is the most common pediatric infratentorial
tumor:). Brainstem gliomas are the third most common pediatric infratentorial
tumor. osborn555. Medulloblastoma the most malignant infratentorial neoplasm in
children, they are second in incidence to cerebellar astrocytoma. kirks125.
1993 Single Best Answer Question
Medulloblastomas (one of the PNET tumors) are one of the most common posterior
fossa masses in the pediatric population, accounting for more than one third of
posterior fossa neoplasms and 50% of cerebellar tumors in children (Grossman &
Yousem, 1994, p. 85) These tumors are usually seen in the midline arising from
the superior velum of the fourth ventricle. Patients usually have hydrocephalus.
On nonenhanced CT the lesions are hyperdense well-circumscribed masses.
Subarachnoid seeding may be present. Borkovich claims that there are slightly
more cerebellar astrocytomas in most series. ???Quoting Lee, Rao, & Zimmerman
1992, p. 389, Depending on the series, medulloblastoma or cerebellar astrocytoma
is the most common primary neoplasm of the
pediatric posterior fossa. According to Grossman and Yousem (1994), pilocytic
astrocytomas are the most common infratentorial neoplasm in children - 20% are in
the brainstem and 40% are cerebellar. Ependymoma is usually associated with the
4th ventricle. They have a greater incidence of calcification (40-50%) than other
posterior fossa pediatric neoplasms. Answer : probably b. medulloblastoma
by KH

133. The most common spinal cord tumor in a child is:

a. hemangioma
b. astrocytoma
c. ependymoma
d. hemangioblastoma

Type- Neuro
Answers- b
Notes-

1993 Single Best Answer Question


Hemangioblastomas account for only 3% of intramedullary spinal cord tumors, and
30% of these are seen with von Hippel-Lindau syndrome. Purely intramedullary
lesions are associated with cyst/syrinx formation in two-thirds of cases - the
presence of a holocord cyst, with only small areas of enhancement, should raise
the possibility of hemangioblastoma. Ependymoma comprises 30% of intramedullary
tumors in a child. They are the most common primary tumors to arise in the lower
spinal cord, conus medularis, and filum terminale. One subtype of ependymoma, the
myxopapillary form, is particularly common in the conus and
filum; it is a mucin-producing tumor, which is prone to bleeding - sometimes
presenting as unexplained subarachnoid hemorrhage. Hemangioma is the most common
benign lesion of the vertebral body in a child. Cobbs syndrome (rare) is
cutaneous malformations, vertebral hemangiomas, and
hemangiomas of the spinal cord. (Kirks p298).
by CM

134. The most like location for an intraventricular Meningioma is:

a. aqueduct of sylvius
b. third ventricle
c. atria of the lateral ventricle

Type- Neuro
Answers- c
Notes-

by KW
Osborn 587,900
Intraventricular meningiomas typically occur around the choroid in the trigone of
the
lateral venticle esp the left lat ventricle, p 68-73 Requisites
2000 Board Question

135. The patient with a dry mouth and bilateral swollen parotid glands most likely
has:

a. AIDS
b. Sjogren syndrome
c. Kaposi sarcoma
d. Lymphoma
e. Lupus

Type- Neuro
Answers- b
Notes-

by KW
Dahnert p437
1997 Old Board Question

136. The reason for pulsatile exopthalmous with neurofibromatosis is?

a. Carotid cavernous fistula


b. Orbital ridge dysplasia
c. Meningioma

Type- Neuro
Answers- b
Notes-

Dahnert p261
In NF-1, pulsitile exopthalmos or unilateral proptosis is caused by herniation of
the subaachnoid space and temporal lobe into the orbit . Harlequin appearance to
the orbit from partial absence of the greater and lesser wing of the sphenoid, and
orbital plate of the frontal bone. (D198)
1998 Board Question
by MH

137. The signal intensity of vertebral bodies on T1 weighted images is best


described as?

a. Hyperintense to intervertebral discs


b. Hypointense to CSF
c. Isointense to intervertebral discs
d. Isointense to the spinal cord
e. Hypointense to intervertebral discs

Type- Neuro
Answers- a
Notes-

T1 of the spine is high because of fatty marrow, the nucleus pulposis is of


relative low signal intensity because it is hydrated. Tumor replacement of the
fatty bone marrow is low signal intensity (Ramsey 2nd ed. 695).
1999 Board Question
by MH

138. There is a single ventricle and midline fusion of the thalami. What is the
most likely diagnosis?

a. holoprosencephaly
b. infarct
c. encephalocele
d. hydrocephalus

Type- Neuro
Answers- a
Notes-

Dahnert p240
1989, 1990 Single Best Answer Question
Meningoencephalocele is a disorder of midline closure commonly seen in the
occipital
region. It represents herniation of the meninges-containing CSF through a
congenital defect in
the skull vault. In an encephalocele, contents include brain tissue. The most
common site is the
occipital region.
by MH

139. There was a simple question regarding the location of the facial nerve in the
IAC:

a. Superior to the cochlear nerve


b. Inferior to cochlear nerve
c. Inferior to crista falciformis

Type- Neuro
Answers- a
Notes-

by KW
2000 Board Question
Dahnert p311
The facial nerve is superior and anterior in the IAC. So the facial nerve is
superior to crista falciformis and anterior to Bills Bar.

140. Thirty-five year old male with back pain. MRI shows a lesion in the
vertebral body that is high signal on T1 and T2. The most likely diagnosis is?

a. Bone island
b. Metastasis
c. Hemangioma
d. Schmorls node

Type- Neuro
Answers- c
Notes-

c. The vertebral bodies and skull are most commonly involved and show dense
vertical striations. High signal intensity on both T1 and T2 (H67).
1996 Question

141. True or False concerning orbital pseudotumor:

a. Responds to steroids
b. Have positive findings on imaging studies
c. Can be painful
d. Is associated with pseudo tumor cerebri
e. Requires radiation treatment

Type- Neuro
Answers- a,b,c
Notes-

by KW
Dahnert p292
Pseudotumor cerebri is bilateral congenital papilladema
If fails steroid therapy, XRT and chemo can be used...
1997 Old Board Question

142. True or False regarding juvenile angiofibromatosis:

a. Is more common in women than men


b. Arises from the nasal cavity
c. Occurs in the pterygopalatine fossa

Type- Neuro
Answers- b,c
Notes-

by KW
Dahnert p325
Almost exclusively in males
Usually adolescents
Most common benign nasopharyngeal tumor
Spreads via the pterygopalatine fossa
1997 Old Board Question

143. True/False regarding neuroradiology:


a. Adenoid cystic carcinoma has a propensity for perineural spread.
b. Squamous cell lung carcinoma is the most likely lung cancer to cavitate.
c. The most common cause of carotid dissection is blunt trauma to the neck.
d. The most common cause of an asymptomatic cystic midline infrahyoid neck
mass is a thyroglossal duct cyst.
e. Most common calcification at the insertion of the optic nerve is drusen.

Type- Neuro
Answers- a,b,c,d,e
Notes-

by KW
Dahnert p314
Adenoid cystic carcinoma = cylindroma, propensity to spread along the facial nerve
4-15% of all salivary tumors
1997 Old Board Question

144. True/False regarding radiographic findings of a meningioma include:

a. Widened middle meningeal grove


b. Widened foramen ovale
c. Widened foramen spinosum
d. Eroded tuberculum sella

Type- Neuro
Answers- a,c,d
Notes-

Dahnert p250
by KW
1997 Old Board Question

145. True/False, A patient with an intracerebral bleed gets acutely worse one week
later. This may be caused by:

a. Re-bleed
b. Communicating hydrocephalus
c. Vasospasm
d. Venous thrombosis

Type- Neuro
Answers- a,c
Notes-

by KW
Dahnert p269
1997 Old Board Question

146. Type II odontoid fracture commonly has non-union?

a. True
b. False

Type- Neuro
Answers- a
Notes-
by KW
Dahnert p169, Resnick p802
The rate of non-union is high in Type II fractures
1997 Old Board Question

147. Typical features of hemangioblastomas in adults include:

a. cystic
b. presents with hemorrhage
c. presents with Parinaud syndrome
d. calcification
e. avascular

Type- Neuro
Answers- a
Notes-

Dahnert p237
1992 True/False Question
Hemangioblastoma is the most common primary neoplasm of the adult posterior fossa
and arise most frequently in the cerebellum. It is associated with von Hippel-
Lindau disease. It
does not typically present with hemorrhage although there may occasionally be
hemorrhage in
the nodular component. They are very vascular with very intense staining on
angiography, and
prominent flow voids on MR. It is solid in one third but commonly appears cystic
+/- mural
nodule. It almost never calcifies. A spinal hemangioblastoma can present with
subarachnoid
hemmorrhage. Common sx include HA, ataxia, N/V, vertigo. 40% can have P vera from
erythropoietin produation. p 81 Requisites
Perinauds syndrome is loss of upward gaze which is seen with pineal tumors.
Reference: Dahnert 1993, pp. 180-181
by MH

148. What are symptoms of an L4 nerve root compression?

a. Lateral thigh pain


b. Decreased ankle relfex
c. Hamstring weakness
d. Decreased patellar reflex
e. Gluteal pain

Type- Neuro
Answers- c,d,e
Notes-

Not Verified
1994 Question

149. What is a carotid-basilar communication?

a. Posterior communicating artery


b. Otic artery

Type- Neuro
Answers- a,b
Notes-

netter133.
Dahnert p208
1989 Board Question
by KH

150. What is the most likely cause of conductive hearing loss and facial nerve
palsy in a trauma patient?

a. vertical fracture of the petrous bone


b. longitudinal fracture of the petrous bone
c. epidural hematoma
d. transverse fracture of the petrous bone

Type- Neuro
Answers- b
Notes-

1992 Single Best Answer Question


Longitudinal fractures (parallel to the long axis) of the petrous bone are
associated with conductive hearing loss secondary to fluid in the middle ear.
Facial nerve (VII) is involved in 10-20% of injuries (this is less common than in
longitudinal fractures.). Transverse fractures (perpendicular to the long axis)
are less common - complications include sensorineural deficits (CN 8) and facial
nerve (VII) injury (40-50%). (D p322)
by CM

151. What is the most likely diagnosis if a 50-60 HU density is seen in the brain
and it has peripheral enhancement?

a. high grade astrocytoma


b. aging hematoma

Type- Neuro
Answers- b
Notes-

Dahnert p234
1991 Single Best Answer Question
by AE

152. What is the normal location of the conus medullaris at birth?

a. T10
b. T12
c. L4
d. S1
e. L2

Type- Neuro
Answers- e
Notes-

In early embryogenesis, the spinal cord passes all the way to the end of the
spinal canal. Each spinal cord segment lies exactly opposite the corresponding
vertebral segment and each segmental nerve root passes laterally to its exit
foramen. As the embryo matures, the vertebral bodies grow faster than the neural
elements, so the spinal cord appears to ascend within the spinal canal. As a
general rule, the tip of the conus medullaris lies at approximately L2-3 at birth
and nearly at the adult level of L1-2 by age three months. The tip of the conus
medullaris lies at or above the L2-3 intervertebral disc space in 97.8% of cases.
It lies over the L-3 vertebra in 1.8% of cases. Therefore, a spinal cord that
lies at or below L-3 is best regarded as abnormal until detailed evaluation proves
it is not held in abnormally low position by a tethering mass, a fibrous band, or
a bone spur. (D p160)
1999 Board Question
by CM

153. What is true about parotid gland tumors?

a. pleomorphic adenoma usually is located deep to the facial nerve


b. warthin tumors are more common in females
c. pleomorphic adenoma is the commonest
d. warthin tumors spread via perineural route

Type- Neuro
Answers- c
Notes-

by KW
Verified Dahnert p335, Som 321-325
Pleomorphic adenoma is most common benign tumor, more common in women greater than
40, is lateral to the facial nerve in greater than 80% of cases
Warthins more common in men
2000 Board Question

154. Where is low density seen by CT with occlusion of the posterior cerebral
artery?

a. Posterior temporal region


b. Contralateral thalamus
c. Ipsilateral thalamus
d. Centrum semiovale
e. Ipsilateral cerebellum

Type- Neuro
Answers- a,b,c
Notes-

Req p.60-62
1989 Board Question
by AE

155. Which are true concerning Arnold Chiari malformations?

a. Occipital menengocele is present in Arnold Chiari malformation type I


b. Tonsilar herniation is present in Arnold Chiari malformation type II
c. Herniation of the fourth ventricle in the vermis is present in Arnold
Chiari malformation type III
d. Vermian hypoplasia is present in Arnold Chiari malformation type IV

Type- Neuro
Answers- b,d
Notes-
by KW
Verified Osborn p24
Type III = Type II + encephalocele
Type IV = cerebellar hypoplasia
1989 Board Question

156. Which is false regarding Dyke Davidoff Mason syndrome or unilateral atrophy:

a. Unilateral decrease in size of cranial fossa


b. Large paranasal sinuses
c. Unilateral thick skull
d. Enlargement of one side ventricles
e. Underpneumatazition of mastoids

Type- Neuro
Answers- e
Notes-

by KW
Dahnert p229
This syndrome is an insult to immature brain resulting in neuronal loss and
impaired brain growth
May be prenatal - congenital malformation, infection, or vascular insult
May be perinatal - birth trauma, anoxia and hypoxia, intracranial hemmorhage
May be postnatal - trauma, tumor, infection, prolonged febrile seizures
The above entail most of the findings, with contraction of a hemisphere or lobe
2000 Board Question

157. Which is most likely to cause acute neurological sequela:

a. Atlanto occipital dislocation


b. Isolate fracture of the posterior arch of C1
c. Hyperextension tear drop

Type- Neuro
Answers- a
Notes-

by KW
Verified Baby Resnick 800, 805, Dahnert p168
Check .... which has worse and more acute neurological sequela....
Both Hyperextension injury and AtlantoOccipital Dislocation share these findings
However, Atlantooccipital dislocation is rarely due to trauma (rheumatoid, downs,
CPPD, etc)
Hyperextension injury is much more common, accounting for 25% of all cervical
trauma cases
Could they have forgot the choice of hyperflexion tear drop, the most sever and
unstable C-Spine injury...
2000 Board Question

158. Which is most often associated with neurological deficit?

a. Hangmans fracture
b. Type II dens fracture
c. Atlanto-occipital disassociation
d. C-1 arch fracture
e. Jefferson fracture
Type- Neuro
Answers- c
Notes-

by KW
Osborne p867
Atlanto-occipital disassociation may have normal xray findings, but is commonly
fatal.
1998 Board Question

159. Which is the most benign feature of thyroid mass?

a. Punctate calcification
b. Multinodularity
c. Peripheral eggshell calcification

Type- Neuro
Answers- b
Notes-

by KW
Ultrasound Requisites p450, Dahnert p304
Peripheral calcification more likely benign, small punctate calcification more
likely malignant
I dont know how periphral calcification compares with multiplicity
A cold nodule in a multinodular thyroid carries a 1-6% risk of carcinoma, compared
to 15-25% for solitary cold nodule
2000 Board Question

160. Which is the most likely cause of a lesion in the cerebellum with a sharp
medial border at the midline?

a. astrocytoma/glioma
b. subacute infarction
c. medulloblastoma
d. metastasis
e. hemangioblastoma

Type- Neuro
Answers- b
Notes-

Dahnert p249
1992 Single Best Answer Question
by MH

161. Which is true regarding parotid gland tumors?

a. pleomorphic adenomas are typically medial to the facial nerve


b. pleomorphic adenoma is more common in women
c. Warthins tumor commonly has perineural spread
d. Warthins tumor is the most common parotid gland neoplasm

Type- Neuro
Answers- b
Notes-

by KW
Verified Dahnert p335, Som 321-325
Pleomorphic adenoma is most common benign tumor, more common in women greater than
40, is lateral to the facial nerve in greater than 80% of cases
Warthins more common in men
1995 True/False Question
80% of tumors are benign, 80% are pleiomorphic adenomas, 80% are in the
superficial
lobe of the parotid, 80% of pleiomorphics are in the parotic, 80% of pleiomorphics
if left unRx
ed stay benign.
The portion of the parotid that extends deep to the plane of the facial nerve is
the deep
lobe. The superficial lobe extends from just under the skin. Thes lobes dont
really exist but are
arbitrary distinction for surgical purpose. If the tumor is in the superficial
lobe, the facial nerve
is dissected out deep to the mass and the tumor is resected superfical to the
facial nerve. Are
most commonly seen in middle aged women. p 414, 420
Warthins tumor are nearly exclusive to the parotid and are the most common
multiple
and bilateral tumors in the gland. These lesions are entirely benign. Warthins
are most
commonly seen in elderly men. p 420 They are the second most common benign tumor
of the
parotic-- 10% as stated in Danhertp 256. These lesions are usually round to oval
encapsulated
masses. p 824 Robbins
ref: Requisites p 420, 414, Danhert p 256, Robbins, p 824

162. Which is true regarding the CT appearance of the following:

a. Anaplastic glioma appears as an isodense, or slightly hyperdense lesion


with enhancing rim
b. A subacute hematoma appears as a thick irregular rim enhancement with a
low density center

Type- Neuro
Answers- b
Notes-

Early subacute hematomas have high signal on T1WI. The high signal typically
begins at teh hematoma periphery and progresses inward. Thus the center of early
subacute clots remains relatively isointense on T1WI and the rim becomes
hyperintense. Early subacute clots are profoundly hypointense on T2WI and
gradient-refocussed sequences. Osborn167.
Anaplastic glioma appears on MR as a poorly delinated lesion that has
heterogeneous signal intensities on both T1 and T2WI. Mixed iso- to hypodense
areas are seen on the T1-weighted sequences. Some hemorrhagic foci may be
present. A common appearance on T2W1 is a central core of hyperintensity
surrounded by an isointense rim with peripheral fingerlike high intensity
projections secondary to vasogenic edema. osborn538
1989 Board Question
by KH

163. Which of the following are associated with increased incidence of primary
brain tumor?
a. Gardners
b. sickle cell disease
c. von Hipple-Lindau
d. Turcots
e. tuberous sclerosis

Type- Neuro
Answers- c,d,e
Notes-

by KW
Osborn 94,104, Dahnert p685,717
Turcots - Glioblastoma (esp. Supratentorials) + Medulloblastoma
von-Hippel - hemangioblastomas
Tuberous Sclerosis - Giant Cell Astrocytomas
1993 True/False Question
Von Hipple-Lindau syndrome is autosomal dominant and consists of cerebellar
hemangioblastomas and retinal angiomatosis (67%), pheochromocytoma, cysts of
kidneys,
pancreas, liver, and epididymis, and a high incidence of renal cell carcinoma.
Turcots is associated with CNS tumors, especially supratentorial glioblastomas.
Gardners consists primarily of colonic polyps, osteomas, and soft tissue tumors.
The intracranial manifestations of tuberus sclerosis include periventricular
subependymal
nodules, cortical and subcortical peripheral tubers, white matter lesions, and
subependymal giant
cell astrocytomas.
In sickle cell disease the incidence of cerebrovascular occlusive disease is 5-
17%.
Infarction is most likely related to stasis and ischemia in the vasa vasorum which
leads to intimal
and medial hyperplasia and subsequent narrowing and occlusion of internal carotid
or proximal
anterior and middle cerebral arteries.

164. Which of the following are causes of intracranial calcifications?

a. hypoparathyroidism
b. tuberous sclerosis
c. progeria
d. Cornelia De Lange
e. Sturge-Weber syndrome

Type- Neuro
Answers- a,b,c,e
Notes-

Dahnert p.48, Cornelia de Lange Syndrome= Amsterdam dwarfism, no mention of


intracranial calcification
Dahnert p.118, Progeria= Hutchinson-Gilford syndrome, autosomal recessive, dwarf,
premature atherosclerosis/vasc. Ca
Differential on Dahnert p.189
1988 True/False Question
by AE

165. Which of the following are causes of third cranial nerve palsy?

a. migraine
b. diabetes
c. multiple sclerosis
d. posterior inferior cerebellar artery aneurysm
e. lymphomatous meningitis

Type- Neuro
Answers- a,b,c,e
Notes-

1994 True/False Question


Diabetes classically gives a pupillary-sparing third-nerve palsy because it does
not
involve the parasympathetic fibers which control pupillary constriction.
An unruptured posterior communicating artery anuerysm classically gives an
isolated and
complete third-nerve palsy with a dilated pupil.
Reference: Atlas, pp. 396-408
by MH

166. Which of the following are frequently calcified lesions in the posterior
fossa?

a. hemangioblastoma
b. dermoid
c. acoustic schwanomma
d. astrocytoma
e. medulloblastoma

Type- Neuro
Answers- b
Notes-

Dermoid- occur in midline of posterior fossa, commonly calcify


Astrocytoma- calcify <10% of the time, in children most often see JPA(good
prognosis), a posterior fossa astrocytoma in an adult is more likely to be a GBM
Medulloblastoma- common post fossa mass in children( not as common as JPA), 10-20%
calcify, compress and distort the fourth ventricle whereas ependymomas enlarge it
Hemangioblastoma- Most common primary post fossa neoplasm in adults( met is most
common mass), calcifications rare, typical findings are a cystic mass with a solid
mural nodule
Acoustic schwanomma- do not calcify, differentiate from meningioma by lack of
dural tail and the fact that shwanommas enlarge the IAC
1993 True/False Question
by AE

167. Which of the following are midline in a child?

a. medulloblastoma
b. ependymoma
c. cerebellar astrocytoma
d. hemangioblastoma

Type- Neuro
Answers- a,b
Notes-

1991 True/False Question


The most typical appearance of a posterior fossa ependymoma on CT is that of an
iso- to hyperdense 4th ventricular mass with punctate calcifications, small cysts,
and moderate contrast enhancement. Nearly 50% will show multifocal, small
calcifications; larger calcifications are occasionally seen, Posterior fossa
astrocytoma is eccentric in location. Hemangioblastoma is hemispheric. In
younger pt with HB, t/c VHL. Medulloblastoma is eccentric in adults in the
posterior fossa. p 82-4 Req. Also in Osborn435
Looks midline to me on Osborn p436... KW
by KH

168. Which of the following are migrational abnormalities?

a. alobar holoprosencephaly
b. schizencephaly
c. Chiari II malformation
d. unilateral megaencephaly
e. heterotopic bands

Type- Neuro
Answers- b,d,e
Notes-

Proliferation - Megalencephaly
Migration - Schiz, Heterotopic bands, pachygyria, polymicrogyria
other migational disorders- pachgyria, polymicrogyria
Megalencephaly is migrational disorder - Neuro requisites. - KW

Kirks 98-106
1995 True/False Question
Schizencephaly is a abnormality of neuronal migration is the 5-7th week. It is a
cleft seen
coursing from the ependyma to the pial surface. The disorder is secondary to
failure of the
germinal matrix to form. p 253
Heterotopia is usually in the gray matter which is located in the wrong place due
to arrest
in neuronal migration. This usually occurs in the 7th to 16th week when migration
of the neuroblasts
form the perivent to the pia is thwarted. Ectopic gray matter is usually in the
subependymal or
subcortical location. p 256
Holoprosencephaly is a congenital abnormality where separation of the right and
left
cerebral hemispheres is incomplete. Alobar type is the most severe and
demonstrates almost no
separation of the cerebral hemispheres and ventricles. There is one large
horseshoe shaped vent
with no interhemispheric fissure. The BG and thalamus are fused. p 252
Etiologies of unilateral cranial enlargement-- Dyke-davidoff-masson syndrom,
hemimegalencephaly(neruonal migrational anomaly), NF, Klippel-Trenaunay Synd,
Proteus
Sydrome. Can get polygyria associated with hemispheric size, or agyria associated
with less sever
hemispheric enlargment. MR demonstrates distorted thickened cortex with
ipsilateral ventricular
dilatation. p 258
Chiari malformation is dx when cerebellar tonsils are below the foramen magnum. ?
no
etiology stated. p 261
ref: Requisites
by AE

169. Which of the following are potential causes of enhancing posterior third
ventricular mass?

a. pinealcytoma
b. pineal cyst
c. vein of Galen aneurysm
d. dysgerminoma
e. PNET

Type- Neuro
Answers- a,b,c,d,e
Notes-

Dahnert p263
1994 True/False Question
Pineal cysts typically ring enhance on MRI. Supratentorial PNETs are distinctly
uncommon and in her tenure at CHOP Jill has not seen one involving the pineal. We
are
therefore not including this. If you include pineoblastomas as a PNET( table 3-6
p 83), the
answer is easy with PNET being true as well.
Pineal region masses include germinoma aka dysgerminona or seminoma which have
marked enhancement and are almost exclusively male, pinealblastoma and
pineocytoma.
Pineocytoma enhance vividly.
Other lesions in this area include pineal and tectal gliomas, cavernous
hemangioma,
meningioma, and benign cyst. Pineal cyst can be large. Pineocytoma can be hard
to differentiate
froma cyst because it can also have cystic components. Key is long-term f/u.
Because pineal
cyst are surrounded by limbs of inernal cerebral veins, have to be careful not to
misread vascular
enhancement as solid mass enhancement.
ref: Requisites p 96-7
by MH

170. Which of the following are true regarding CSF rhinorrhea?

a. medial blow-out fracture of the orbit is a cause


b. most commonly involves the sphenoid sinus
c. temporal lobe arachnoid cyst can be a cause
d. mucus retention cyst of the frontal sinus can be a cause
e. osteoma of the frontal sinus can be a cause

Type- Neuro
Answers- e
Notes-

1993 True/False Question


An osteoma is a bony mass which occasionally results in mucocele and/or
pneumocephalus as the posterior wall of the frontal sinus is breached (Som p201.
A medial wall blow-out fracture involves the lamina papyracea of the ethmoid bone.
A depression fracture of the orbital roof may case CSF rhinorrhea(Head & Neck
Imaging-Som p241-244).
Traumatic CSF rhinorrhea most commonly is secondary to fractures through the
frontoethmoidal complex and middle cranial fossa. The most common cause of CSF
rhinorrhea is trauma.
Reference: Dahnert 1993, p. 124, 127
Can arise from postoperatively when the cribiform plate is violated. Usually get
post ethmoidectomy p 368 Requisites.
Pneumocephalous is found in 20% of pt with CSF leaks. Blood clot forms around the
fx initially, but when the clot resolves, CSF leak begins. Usually the CSF leaks
stops within the 1st week, 99% w/in 6wks. Post trauma CSF leaks most often involve
the frontal, ethmoid sinus, and cribiform plate. Less frequently, the sphenoid
sinus, petrous and other calvarial sites are involved. p516 Lee, Rao, Zimm.
Internal orbital wall blowout involves fx of ethmoid or maxilllary sinus Medial
wall fx are characterized by the passage of air thru the fx defect into the orbit
giving orbital emphysema. The air gains access w/ increased intranasal pressure
such as nose blowing. Medial blow out fx characteristically involves the medial
floor and the medial orbital wall, and medial wall of the maxillary antrum.
Clinical signs of blow out include enopthalomos, diploplia, limited upward and
outward gaze.p 51 Harris and Harris
d. (Som p128,248) e. (D p218)
by CM

171. Which of the following are true regarding herpes encephalitis?

a. patients are most often immunocompromised


b. more commonly seen in children than adults
c. involves the temporal lobes
d. often hemorrhagic
e. due to Herpes zoster virus

Type- Neuro
Answers- c,d
Notes-

HSV causes a fulminant hemorrhagic, necrotizing meningoencephalitis. Severe edema


and massive tissue necrosis with petechial and some confluent hemorrhages are
typical findings.Infection is localized to the temporal lobes, insular cortex,
subfrontal area, and the cingulate gyri. Osborn694
1992 True/False Question
Herpes encephalitis is more common in adults. In children it is secondary to HSV
II which can cause a variety of findings: calcification, microcephaly,
microphthalmia, retinal dysplasia, and encephalitis, as well as severe sequellae
in survivors. HSV I is the most common cause of herpes encephalitis in adults.
It usually involves the temporal lobes asymmetrically, the insula, the
orbitofrontal region, and the cingulate gyrus. One third is primary and 2/3 is
reactivation. There is often hemorrhage (either gross or petechial). Zoster
causes a granulomatous angiitis in immunocompromised patients. Reference:
Grossman and Yousem 1994, pp. 182-183
by KH

172. Which of the following are true regarding the position of the facial nerve in
the internal auditory canal?

a. anterior to the vestibular nerve


b. above the crista falciformis
c. superior to the cochlear nerve
d. below the crista falciformis

Type- Neuro
Answers- b,c
Notes-

by KW
Dahnert p311
1988 True/False Question
The facial nerve is superior and anterior in the IAC. So the facial nerve is
superior to crista falciformis.

173. Which of the following are true with regards to amyloid angiopathy?

a. associated with hemorrhage in the cerebral hemisphere


b. is seen most commonly in people with systemic hypertension
c. is associated with hemorrhage of the basal ganglia
d. associated with hemorrhage of the brainstem
e. most commonly seen in the elderly

Type- Neuro
Answers- a,e
Notes-

1993 True/False Question


Amyloid angiopathy = Deposition of amyloid in the media and adventitia of small
and
medium-sized vessels of the superficial layers of the cerebral cortex and
leptomeninges with
sparing of the deep gray nuclei. Amyloid accumulates Congo red dye and exhibits
yellow-green
birefringence when examined with polarized light. Hemorrhages are usually lobar,
involving
frontal and parietal lobes including subjacent white matter. Subarachnoid and
subdural
hemorrhages have also been reported. There is a propensity for recurrent
hemorrhages or
multiple simultaneous hemorrhages. The hemorrhages are rarely found in the
cerebellum, white
matter, basal ganglia, and brain stem. The major risk factor is aging, but doesnt
correlate with
hypertension. It is not associated with systemic amyloidosis.
Reference: Grossman & Yousem 1994, pp. 132-133
by MH

174. Which of the following best describes the appearance of the pituitary gland
on MR imaging?

a. Typically dark on T2 sequences


b. Demonstrates early and prominent enhancement
c. Typical appears flat or with a mild concave superior surface
d. Enhances less than an adenoma

Type- Neuro
Answers- b,c
Notes-

Dahnert p202
Isointense on T1/T2 to white matter
1999 Board Question
by AE
175. Which of the following can most closely resemble MS on MRI of the brain?

a. Lymphoma
b. Alzheimers disease
c. Acute infarct
d. Lyme disease
e. Herpes encephalitis

Type- Neuro
Answers- d
Notes-

A.MS is a white matter disease with ovoid periventricular lesions oriented


perpendicularly to the long axis of the brain and lateral ventricles. The second
most common site is the corpus callosum 50-90%. Most authorities require the
presence of three or more discrete lesions that are 5mm or greater in size in
characteristic locations (Dawsons fingers). (O756-7)
Lyme disease is caused by the tick-borne spirochete Borrelia Burgdorferi, most
commonly in New England, the Pacific states, and Wisconsin, and causes neurologic
complications in 10-15% with cranial nerve palsies and peripheral neuropathies.
Imaging reveals superficial and deep white matter changes that may enhance.
Imaging is indistinguishable from MS or other postviral encephalidities such as
ADEM. (O706)
Alzheimers disease show diffuse cortical atrophy with widened sulci and enlarged
lateral ventricles (gray matter). The temporal lobes are disproportionately
affected, particularly the hippocampal formations. (O772)
Herpes encephalitis is usually caused by HSV-2 in neonates, and by HSV-1 in
children and adults. During oral herpes infection, the virus is transported along
the sensory nerve fibers to the olfactory nerve or gasserian ganglion.
Encephalitis results from retrograde viral spread from the trigeminal ganglion in
the primary infection, reinfection, or activation of latent infection. Has a
prediliction for the limbic system (temporal lobes, cingulated gyri, subfrontal
region). MR shows gyral edema in the temporal lobes. Hemorrhage is uncommon
early. (O694-5)
Primary CNS lymphoma occurs in immunologically normal and abnormal (AIDS)
patients. MR can show diffuse or focal involvement. Most focal PCLs are
isointense to slightly hypointense compared to gray matter on T1WI, and are iso-
to slightly hyperintense on T2WI. AIDS PCL is heterogeneous with hemorrhage and
necrotic foci. The diffuse form can affect the deep gray matter nuclei and white
matter tracts. T2WI show extensive but poorly delineated hyperintensities in the
pons, cerebellum, cerebral white matter, and the basal ganglia. (O620-2)
1998 Board Question
by AE

176. Which of the following cause a cystic midline mass that can mimic an enlarged
third or fourth ventricle?

a. dermoid cyst
b. suprasellar arachnoid cyst
c. intraventricular cysticercosis
d. cerebellar astrocytoma
e. hypothalamic polycystic astrocytoma

Type- Neuro
Answers- b,c,d
Notes-
1993 True/False Question
Im guessing on the correct answers here.
A dermoid cyst is a midline mass which may occur in the 4th ventricle(Osborn p67,
D p228-9). Its capsule is thick, commonly with peripheral calcification (p 320
Requisites). The presence of fat or calcium in it can distinguish it from a large
ventricle.
Cysticercosis is one of the most common parasitic diseases that affect the brain.
A large cyst or cluster of cysts within the 4th ventricle may simulate dilatation
of that ventricle, Lee, Rao, and Zimmerman 1992, pp. 572-573. (Dahnert p227)
Cysticercosis is acquired by ingestion of a parasite which is in insufficiently
cooked pork. The cysticerci are located in brain parenchyma, subarachnoid space,
and the ventricles (rarely intraspinal). The parenchymal cysts have a propensity
for cortical and deep gray matter; whereas subarachnoid cysts can produce
hydrocephalus and mass lesions, particularly in the suprasellar cistern,
cerebellopontine angle cistern, and the Sylvian cistern. There are four stages of
the cyst. In stage I, the larvae are alive and the cyst has clear fluid. Edema
is minimal and the cyst is surrounded by a thin capsule. On MR, the fluid of the
cyst is isointense on all pulse sequences and the eccentric scolex (which appears
as a mural nodule) can be identified. CT shows a circumscribed cyst with density
similar to CSF (the scolex is more dense). Visualization of the scolex is
pathognomonic of cysticercosis. When the larvae die, the cysts incite an
inflammatory response.
The hypothalamic polycystic astrocytoma has incomplete necrosis which sometimes
leads to a polycystic appearance. Supratentorial astrocytomas in children
frequently involve the thalamus. (D p235)
If a suprasellar arachnoid cyst invaginates into the 3rd ventricle, it can be
mistaken for an enlarged 3rd ventricle on the basis of acqueductal stenosis.
Most cerebellar astrocytomas arise in the midline and can be cystic. Cystic
astrocytomas with a mural nodule account for 50% of cerebellar astrocytomas in
children(Dahnert p222).
d.(Osborn p481)
by CM

177. Which of the following causes gyriform enhancement?

a. Sturge-Weber syndrome
b. One day old infarct
c. Ten day old infarct
d. Twenty day old infarct
e. Immediately post seizure

Type- Neuro
Answers- a,c
Notes-

by KW
Verified Osborn 100, 347, Dahnert p193
1989 Board Question

178. Which of the following commonly cause hemorrhagic metastases in the brain?

a. choriocarcinoma
b. melanoma
c. colon
d. lymphoma
e. renal cell carcinoma

Type- Neuro
Answers- a,b,e
Notes-

Dahnert p253
Also oat cell carcinoma
1991 True/False Question
Mets which have a propensity for hemmorrhage include melanoma, renal cell,
choriocarcinoma, and thyroid ca. While bronchogenic ca doesnt have a propensity
for
hemorrhage, hemorrhagic lesions may be seen b/c lung ca is so common.
ref: p 423 Lee, Rao, Zimmerman- Cranial MR and CT
by MH

179. Which of the following exhibit dramatic clinical response to steroids?

a. herpes encephalitis
b. multiple sclerosis
c. cytotoxic edema secondary to infarct
d. CNS lymphoma
e. vasogenic edema secondary to tumor

Type- Neuro
Answers- b,d,e
Notes-

Dahnert p255
1994 True/False Question
Vasogenic edema results from breakdown of the blood brain varrier. Cytotoxic
edema begins as intracellular swelling secondry to any type of insult to the cell
membrane usually due to ischemia p 295 Lee, Rao
In stroke, treatment of edema is done with water restriction and agents that raise
serum osmolality such as mannitol. p 1996 With edema secondary to tumor,
treatment is based on lowering intracranial pressure. Treatment with daily doses
of dexamethasone or methylprednisone reduces cerebral edema and improves surgical
prognosis. This may not control sx of obstructed ventricular system, so a
ventriculostomy may need to be placed. p 2014 HSV encephalities is Rx with IV
antiviral therapy- acyclovir which has been noted to shorten the course of
infection. p 684 CNS lymphoma is Rxed with high dose methotrexate,
glucocorticoids alone or in combo with cyclophosphamide, doxorubicin, vincristine
followed by XRT. p 2019 No effective treatment for MS is known. Therapy is
directed towards reducing the acute episode, prevent relapses or progression, and
relief of Sx. In acute flare, glucocorticoid treatment may lessen the severity,
but recoverty is not improved or the extent of disability is not altered. Can give
ACTH. Immunosupressive agents like azathioprine, and cyclophosphamide have
claimed to reduce the # of relapses but there is no consensus. All tried is total
lymphoid radiation, alpha and beta interferon, diet alteration. p 2042-3 ref:
Harrisions 12th ed
by MH

180. Which of the following injuries to the cervical spine is most likely to cause
neurologic symptoms?

a. Extension teardrop fracture


b. Odontoid fracture
c. Hangmans fracture
d. Jefferson fracture
e. Flexion teardrop fracture
Type- Neuro
Answers- e
Notes-

1999 Board Question


by KH

181. Which of the following is a contraindication for performing a MRI:

a. hip prostheses
b. Stuart Edwards valve
c. metallic foreign body within the eye
d. Greenfield filter

Type- Neuro
Answers- c
Notes-

by KW
2000 Board Question
Consensus...
Absolute contraindication means that the patient absolutely should not be scanned.
This includes cardiac pacemakers, automatic internal defribrillators, implanted
infusion pumps, implanted insulin pumps, bone growth stimulators, non-removable
neurological stimulators, cochlear implants, metal in the eyes, shrapnel in vital
locations, and tattooed eyeliners or lipsticks.
Relative contraindication means that some of these devices may and some may not be
scanned. This includes aneurysm clips, penile prostheses, cardiac valve
prostheses, middle ear prostheses, and shrapnel or foreign bodies in non-vital
locations. Safe means that the device may be scanned without any problems. This
typically includes orthopedic prostheses, pins, rods, plates, surgical clips (2-3
months post surgery), dental fillings, orthodontics, braces, root canal work,
intrauterine devices, and contraceptive diaphragms. Controversial means that there
is no general consensus on the safety of the device. This includes such things as
Gianturco embolization coils, inferior vena cava filters, and endovascular stents.
Dr. Frank Shellock, who has done a great deal of research regarding the MR
compatability of various implants, cautions that his list cannot be taken as the
final word. It is prudent to assume that no list of supposedly MR compatible
devices is absolutely correct. Paraphrasing Dr. Shellock from his latest pocket
guide on MR procedures, ... a manufacturer may change the composition of the
implant, material, device or object without going back to the FDA for new approval
as long as the function of the device remains the same ... Of course the final
decision, when there is a question of safety, is the responsibility of the
physician.
Contraindicacions of MRI exam:
-Pacemakers, defibrillators or neurostimulators
-CNS surgical clips
-Ocular implants or foreign bodies
-Metallic fragments near a vital structure
-Othologic implants or hearing aids
-Starr-Edwards mitral prosthetic heart valve pre-6000 series
-Claustrophobia
-Thermodilution catheters, pulse oximeters, standard ECG wires and leads
-Pregnancy (controversial)

182. Which of the following is associated with the shaken baby syndrome?

a. retinal hemorrhage
b. white matter shear injury
c. cortical hematoma
d. subdural hematoma
e. Schizencephaly

Type- Neuro
Answers- a,b,c,d
Notes-

In others we did not include white matter shear injury.... true or false?
by KW
Verified Dahnert p41, Osborn 244-5
1993 True/False Question
In shaken baby syndrome retinal hemorrhages are observed with subdural
(particularly interhemispheric) hematomas and/or subarachnoid hemorrhage.
Contusions and diffuse cerebral swelling are also noted. The abused child can
have epidural, subdural, subarachnoid, and intraventricular hemorrhages as well as
DAI. Traumatic subdural hematomas in the child (from any traumatic cause) are
bilateral in 80%.

183. Which of the following is bright on T1 and dark on T2?

a. Extracellular methemoglobin
b. Intracellular methemoglobin
c. Deoxyhemoglobin
d. Oxyhemoglobin
e. Hemosiderin

Type- Neuro
Answers- b
Notes-

D239
1999 Board Question
by KH

184. Which of the following is involved in otosclerosis?

a. cochlea
b. stapedius
c. vestible
d. malleolus

Type- Neuro
Answers- a,b
Notes-

D329, stapedial=fenestral otosclerosis (80-90%): Cochlear=retrofenestral


otosclerosis (10-20%)
1994, 1988 Single Best Answer Question
Otosclerosis (Som and Berg 1038) (aka otospongiosis) represents the deposition of
spongy vascular new bone within the otic capsule. It is another cause of
sensorineural hearing loss that is usually bilateral and seen most frequently in
young to middle aged women. In the early stages one identifies a lytic lucent
erosion of the labyrinthine margins of the oval window, the round window niche,
and the cochlea. The cochlea is therefore the best choice. The middle and baseal
turns are most frequently involvoed showing areas of demineralization.
The acoustic pathway is divided into sensory (cochlear) and neural (retrocochlear)
components. Congenital or inflammatory lesions of the cochlea (cochlear
otosclerosis) cause a sensory hearing loss. Retrocochlear hearing loss has
multiple etiologies; acoustic schwannoma is the classic tumor responsible for
neural hearing loss.
p 351 Req
by MH

185. Which of the following is least likely to be a posterior fossa tumor in a


child?

a. astrocytoma
b. choroid plexus papilloma
c. PNET
d. ependymoma
e. brainstem glioma

Type- Neuro
Answers- b
Notes-

1994, 1992, 1988 Single Best Answer Question


The most common posterior fossa lesions in kids are pilocytic astrocytoma and PNET
(medulloblastoma) which each account for 1/4-1/3. Brainstem glioma and
ependymomas are the next most common (each about 1/6). While choroid plexus
papillomas occur most frequently in
kids (35% of cases occur in kids less than 5 y/o) and are in the 4th ventricle,
they account for only 3% of childhood tumors. (Grossman and Yousem 1994, p. 78)
From Jill and Josh:
Medulloblastoma (30-40% of posterior fossa neoplasms in children (Barkovtch, p.
324)) and cerebellar astrocytoma are the most common. Ependymoma represents 15%
of posterior fossa neoplasms (Barkovich, p.334). Osborne states that choroid
plexus papilloma almost
always arises in the lateral ventricles in children (In adults, the 4th ventricle
is the most common location.). These are uncommon tumors with a median age of 26
months. Symptoms are generally due to hydrocephalus. The answer most likely is
choroid plexus papilloma.
by AE

186. Which of the following is most highly associated with peripheral aneurysms of
the MCA?

a. Berry aneurysm
b. mycotic aneurysm
c. Takayasus arteritis
d. polyarteritis nodosa

Type- Neuro
Answers- b
Notes-

by KW
Dahnert p217,533, Osborn 272
Polyarteritis nodosa can have similiar picture, but is rare, and is more
associated with renal aneurysms
1993, 1988 Single Best Answer Question
A mycotic aneurysm is often secondary to endocarditis with septic emboli to the
vasa vasorum and secondary destruction of the vessel wall - all that is left is
the intima. The aneurysms may be saccular or fusiform. The aneurysms tend to be
peripheral in the MCA
distribution.
Polyarteritis nodosa is a multisystem disease with necrotizing inflammation of
muscular arteries. There is CNS involvement in 45% (usually late). There are no
specific findings.
Berry anuerysms, or saccular aneurysms, are congenital weaknesses in the media and
elastica of the arterial wall. The occur at branching point where the parent
vessel is curving.
Takayasus arteritis tends to involve large vessels coming off the arch. Aneurysms
are not a typical feature. Generalized vasculitis may lead to infarcts.

187. Which of the following is not associated with tethered cord and spinal
dysraphism?

a. thickening of the filum terminale


b. diastematomyelia
c. myolipoma
d. hydrosyringomyelia
e. myelolipoma

Type- Neuro
Answers- e
Notes-

by KW
Osborn p807-808, Dahnert p179
Trick question i think
Dorlands Dictionary, p1089 and 1093:
Myelolipoma = A rare benign tumor of the adrenal gland, composed of adipose
tissue, lymphocytes, and myeloid cells
Myolipoma = A benign mesenchymoma containing fatty or lipomatouis elements
Note that the text below is wrong concerning myelolipoma.
1993, 1990, 1989 Single Best Answer Question
Spinal dysraphism basically means a group of spinal anomalies with the common
feature of imperfect union of midline structures. Tethered cord is a spinal cord
that is held in an abnormally low position by a fibrous band, bone spur, or mass.
Diastematomyelia is a longitudinal split in the spinal cord, conus medullaris, or
filum terminale associated with bony abnormalities including spina bifida. It
also is associated with tethering of the conus medullaris (below L2 in 76%).
Hydrosyringomyelia is a cavity in the spinal cord. Most are associated with
congenital spinal and hindbrain abnormalities. Thickening of the filum terminale
(= tethered spinal cord syndrome) is associated with
midline bony defects. Myelolipoma is lipoma(s) of the filum terminale. This is
another form of spinal dysraphism and tethering of the spinal cord. These lesions
have fat on CT or MR. Spinal lipomas are masses of fat and connective tissue that
are at least partially encapsulated and have definite connection to the spinal
cord and meninges. They are considered in three major groups: intradural lipomas,
lipomyelomeninigoceles, and lipomas of the filum terminale. In patients with
lipomyelocele there is no substantial meningocele component and the lipoma usually
extends into the canal through an occult spina bifida. The lipoma is attached to
the dorsal surface of the neural tissue and is covered by intact skin. The spinal
cord may assume different shapes and associated tethering may be present.
Fibrolipomas frequently occur in the filum terminale, and may be considered a
normal variant if not associated with tethering of the spinal cord or neurologic
dysfunction. These developmental lesions arise secondary to the persistence of
caudally located pleuripotential cells that differentiate into fat. Fibrolipomas
of the filum may be intradural or extradural, or both; extradural lesions tend to
be diffuse.
188. Which of the following is the most common cause of a peripheral MCA
microaneurysm?

a. intravenous amphetamine
b. berry aneurysm
c. atherosclerosis
d. septic emboli
e. polyarteritis nodosa

Type- Neuro
Answers- d
Notes-

Dahnert p217
1993 Single Best Answer Question
Mycotic aneurysms are peripheral to the 1st bifurcation of a major vessel in 64%
and are often located near the surface of the brain, especially over the
convexities. (e.g., often just distal
to MCA trifurcation in the inferolateral Sylvian fissure.)
Polyarteritis nodosa may involve all organs, but mainly the kidney (85%), heart
(65%), and liver (50%).
Intravenous amphetamine can yield a CNS vasculitis and subsequent microaneurysms,
but this is rare.
by MH

189. Which of the following is the most common CNS infection in AIDS?

a. TB
b. Cryptococcus
c. HIV encephalitis
d. Toxoplasmosis
e. Histoplasmosis

Type- Neuro
Answers- c
Notes-

Osborne p.698
A.HIV is a neurotrophic virus that can directly involve both the peripheral and
central nervous systems. The HIV virus itself is the most common CNS pathogen in
AIDS patients, followed by Toxoplasma gondi, and Cryptococcus neoformans. (O696)
1998 Board Question
by AE

190. Which of the following is the most common posterior fossa tumor in an adult?

a. hemangioblastoma
b. metastasis
c. medulloblastoma
d. glioma
e. ependymoma

Type- Neuro
Answers- b
Notes-

1992 Single Best Answer Question


A metastasis is the most common posterior fossa tumor in an adult (Grossman and
Yousen 1994, p. 79). However, infarct is the most common posterior fossa mass in
an adult. Lung and breast are the most common metastatic sources (followed by
melanoma, RCC, and thyroid). Hemangioblastoma is the most common primary tumor of
the posterior fossa on an adult.
by MH

191. Which of the following is the most common supratentorial neoplasm in a child?

a. meningioma
b. ependymoma
c. glioma
d. PNET
e. hemangioblastoma

Type- Neuro
Answers- c
Notes-

1994 Single Best Answer Question


Gliomas constitute 30% of supratentorial neoplasm in children according to
Barkovich on page 361. He further states (on page 368) that supratentorial
ependymomas constitute 20-40% of childhood ependymomas. PNETs constitute 5% of
childhood supratentorial neoplasms. The
answer is most likely supratentorial gliomas.
by AE

192. Which of the following is the most convincing evidence against multiple
sclerosis in a patient?

a. one neurologic event, normal physical exam


b. normal MR of the brain
c. internuclear ophthalmoplegia

Type- Neuro
Answers- a
Notes-

by KW
Dahnert p254
The following suggest a process other than MS-
No eye findings
No bladder findings
No sensory findings
No clinical remission
Totally local disease
1991 Single Best Answer Question
Multiple sclerosis is a demyelinating disease of unknown etiology which most
frequently
affects young adults. The classic lesions seen on MR are plaques, which occur
commonly in the
periventricular white matter, internal capsule, corpus callosum, pons, and
brachium pontis. They
are best seen on long TR sequences.

193. Which of the following is the most likely cause of an enhancing suprasellar
mass in a 1-year-old?
a. pituitary adenoma
b. glioma
c. neuroblastoma metastasis
d. dysgerminoma
e. craniopharyngioma

Type- Neuro
Answers- e
Notes-

1995 Single Best Answer Question


CNS tumors account for more deaths during childhood than any other malignancy
except leukemia. Primary intracranial neoplasm accounts for 15% of neoplastic
disease during childhood.
Supratentorial primary tumors include cerebral astrocytoma (37%), optic glioma
(13%),craniopharyngioma (12%), ependymoma (5%) and miscellaneous. p 133 Kirks
ref: Requisites p 320-6. Kirks p130-133, Requisites p97 In children, the most
common lesions in the suprasellar region are chaniopharyngioma and hypothalamic-
opticochiasmatic glioma. Craniopharyngiomas account for half of all suprasellar
tumors in children. Astrocytomas of the visual pathway, optic nerve, chiasm, and
optic tracts account for25-30% of pediatric suprasellar neoplasms.(Osborn
p472,475,476)
by CM

194. Which of the following is the most likely cause of conductive hearing loss
and facial nerve palsy in a patient with acute trauma?

a. temporal lobe hematoma


b. epidural hematoma
c. transverse fracture of the temporal bone
d. longitudinal fracture of the temporal bone

Type- Neuro
Answers- d
Notes-

Can also read in D322.


1994, 1992, 1988 Single Best Answer Question
palsy typically associated with a transverse fracture of the temporal bone -
transverse fractures are only one fifth as common as longitudinal fractures and
are perpendicular to the petrous ridge.
Smirniotopoulos indicates that sensorineural hearing loss due to both parts of CN
VIII being affected is a complication. Facial nerve can be involved in both but
more typically in transverse fractures (40-50%).
Longitudinal fractures usually spare the inner ear with disruption of the
ossicular chain. Longitudinal fractures are parallel to the petrous ridge. The
facial nerve is affected in 10-20%, but the eighth cranial nerve typically remains
intact, resulting in a conductive hearing loss. These represent the classical
teaching and like all other things is far from the reality with oblique fractures
being the most common. (Also, see question 34 regarding fractures of the petrous
bone.) References: AFIP notes (Smirniotopoulos); Seminars in US, CT, and MR
10:262-279, 1989; Latchaw, 2nd edition, 1991, pp. 178-179, 203-265; Harnsberger,
pp. 325-326; Barkovich, pp. 68-71
by KH

Вам также может понравиться